Tuesday, June 27, 2017

It’s the next open thread


Here is your latest opportunity to converse about topics that have not arisen in the course of other combox discussions at this here blog.  From neo-Kantianism to neo-conservatism, from mortal sin to imported gin, from the dubia cardinals to the Doobie Brothers – discuss whatever you like, within reason.  Keep it civil, but for once you needn’t keep it on topic.

By the way, you need not address comments or questions to me personally, because more likely than not I will not have time to respond.  This is not an “Ask Ed anything” post, sorry.  Talk amongst yourselves.

221 comments:

  1. Two questions:
    1. How would you show that act and potency are real features of the world?
    I'd like to argue that given the principle of non-contradiction (so here, no change will bring itself about by existing and having causal power before it exists) and given the reality of change, they are unavoidable. What else do I need?
    2. On free will, I'd like to argue for a sort of compatibilism that has the first mover argument accounting for all change in a deterministic way, but with secondary movers of a specific sort, that is rational animals with an immaterial soul, able to exercise choice in accepting or not divine premotion. Does this sound as if it's on the right lines?
    Thanks for any help!

    ReplyDelete
  2. Hi, Chris Giles. With regards to your second argument on free will, I would say that you really don't have to use the label "compatibilism," since a libertarian could accept the same argument, as far as I can see.

    ReplyDelete
  3. Does anyone here know of any orthodox and funded MA (or graduate certificate) in Theology programs? I'm trying to get prerequisites done so I can pursue studies in canon law. Any help is appreciated!

    ReplyDelete
    Replies
    1. Hey Christian, I'm not sure as to how good the funding is, but I would rank the Dominican School of Philosophy and Theology in Berkeley, CA, high on the orthodoxy list--and this especially for a fan of Feser's work. The faculty and student body are first-rate in every relevant sense.

      Delete
    2. Dr. Feser,

      What in your opinion is the best argument against teleology? The fact that natural objects always behave a certain way seems as obvious as ever. At least Descartes, Paley, and Co. opted for a derived teleology. I could understand making that mistake. However there are plenty of bright atheists who practically worship science (which is essentially studying final and efficient causality) while rejecting final causality. It seems like an obvious contradiction. All atheists cannot be this stupid, so I feel like I am missing something. Do they suppose a many worlds hypothesis in order to account for the "fortuitousness" of natural phenomena, or perhaps they prefer to appeal to brute facts? I would look on atheist blogs, but I typically read garbled nonsense that misses the point and actually has the reverse effect of helping me understand their viewpoints. Any atheist philosophers you could recommend?

      Thank you for the help.

      Delete
  4. One issue I think is interesting is how the reality of time could be demonstrated, and whether the A theory (or common sense) view of time (namely that time is a real feature of the world which moves from past to future) is necessary for the distinction of act and potency

    ReplyDelete
    Replies
    1. The A-theory in my view is not really the "common sense" view, even though it perhaps appears that way at first glance.

      Presentism (and the A-theory which it is closely associated with) is deeply problematic for a number of reasons, philosophical, theological, and scientific.

      I'd point out that if presentism is true, it is incredibly difficult to account for how God knows the future and for what could ground truths about the past.

      I think however that the Act-potency distinction is compatible with the B-theory of time. The B-theory does not deny change nor does it undermine modality which provides a distinct motivation for the act/potency distinction.

      Tom

      Delete
    2. From my reading, I think a majority of Thomists are A-theorists, but there is no reason that you have to be an A-theorist and a Thomist. Like Tom said, Thomists want to affirm change and modality, and b-theory by itself does not deny this. Jeffrey Brower has a good book called Aquinas's Ontology of the Material World that deals with act/potency from a non presentist perspective that should help you out.

      Delete
    3. CZ, I would put it slightly differently: I suspect that neither the A-theory nor the B-theory, in their standard basic forms, are valid simply speaking. I have reservations about both, and suspect that at least one of them can be modified to fit more suitably to the act-potency / change principles of A-T.

      Not that I claim to have done it. B-theory at present suffers from the fact that all of our language use verbs with tense built in: it needs a tenseless language form (that still includes different times) to be stated properly . Let me know when they get one.

      Delete
    4. Tom wrote: ``I'd point out that if presentism is true, it is incredibly difficult to account for how God knows the future and for what could ground truths about the past.´´
      I was never able to reconcile belief in free will with the traditional concept of omniscience, nor with the B theory, aka the block model. I can't believe in (incompatibilist) free will while simultaneously believing there are true propositions about my future. But maybe that just reflects my intellectual limitations. In any case, when I was a theist I embraced the process version wherein temporal process is an irreducible feature of reality, even for God, who, like us, faces an open or indeterminate future.

      When I read Feser's `The Last Superstition´ I got the impression that Thomism fits with the B model, but I can't recall that he specifically addressed the nature of time.

      Sogn

      Delete
    5. Sogn,

      How would a libertarian model of free will be undermined by the B-theory?

      Simply because the future exists, and there are true propositions (tenseless) about it, does not mean the future is determined by the past. If we define determinism as 'an event is determined if and only if there is some factor prior to the event which causally necessitates the event' then the B-theory certainly doesn't entail determinism. For one, propositions reporting events which to us are future, are not causally efficacious and cannot determine anything.

      Moreover, positing the existence of the future, as eternalists do, does not tell us whether or not the laws of physics are deterministic or probabilistic. One therefore can consistently maintain that the future exists while at the same time denying that there are any factors in the present which causally necessitate that future.

      It seems to me that if we say eternalism is incompatible with libertarian freedom, then we ought to say that libertarian freedom is an impossibility (which of course, some do argue). The reason is that even the presentist admits that the present exists. Therefore, if the existence of an event entails that the event is not free in an incompatibilst sense, it follows that nobody presently acts free in an incompatibilst sense. But if nobody presently is free in an incompatibilst sense, then no one is free in this way.

      Tom

      Delete
    6. Interesting discussion, the only problem that I see here is that I've never been able to find a clear definition of what Thomists mean by change,or like what do they mean when they say for example, X changes?

      Delete
    7. Has anyone ever done a study of the Scholastic distinction between God's eternity, man's participation in God's eternity, and the temporality of the created world? The only scholar I'm aware of who has pointed this out is J. Glenn Friesen (jgfriesen.wordpress.com), who is much more interested in "Calvinistic" philosophy than the Scholastics *per se*.

      Delete
    8. I only make the above comments to question whether Thomists and Scholastics in general still consider the tripartite distinction (eternity-aevum-time) to be vaild. Henry of Ghent thought so: https://www.amazon.com/Questions-Thirty-One-Thirty-Two-Philosophical-Translation/

      Delete
    9. Tom,

      I agree that the existence of future true/false propositions doesn't entail determinism. I agree in particular that the B-theory doesn't entail determinism. The issue for me is solely whether the future is determinate (fixed, definite) regardless of whether it's causally determined. My problem is that if my entire existence in every detail from conception to death is timelessly/tenselessly/eternally fixed, then my belief in multiple objectively real possibilities is a delusion. But I believe that free will entails the objective reality of alternative possibilities. When I deliberate about a course of action I cannot help but assume that two or more different life paths or trajectories are genuinely open to me depending on the action I choose. But if that's so, then there can be no true future propositions describing the outcome of my decision. My belief that I'm metaphysically free, as opposed to merely uncoerced, is incompatible with a belief that events proceeding from one course of action are eternally "located" at certain immutable spacetime coordinates.

      Again this is not necessarily tied to the issue of determinism. I'm incapable of combining belief in free will with belief in traditional omniscience, or with the B-theory of time, no less than with determinism. I do not believe that propositions reporting future events are causally efficacious or capable of determining anything. I agree that one "can consistently maintain that the future exists while at the same time denying that there are any factors in the present which causally necessitate that future." I take issue not with causality, but simply with whether my future is closed, _regardless_ of the reason why it's closed, e.g. due to the timeless knowledge of God.

      I'm sorry, but you will have to rephrase or elaborate on your last paragraph. I can't see why the existence of a present event presents the same problem as the existence of future events.

      I certainly appreciate your reply.

      Sogn

      Delete
    10. Sogn,

      I'd start by pointing out that on standard understandings of incompatibilism, provided that determinism is false and our actions are up to us in some suitable way, then it follows that we have free will.

      Now regarding alternative possibilities, you say "if my entire existence in every detail ... fixed, then my belief in multiple ... possibilities is a delusion."

      But I would disagree with this. Consider the choice Joe makes on 1/1 2020 to marry Laura. This choice, according to the eternalist, exists. But is it possible that Joe choose to not marry Laura? According to standard possible world semantics, then yes. There is a possible world in which Joe does not marry Laura. Moreover, according to the libertarian, there is a possible world exactly like the actual world up until 1/1 2020 in which Joe does not marry Laura.

      It is tempting to say that if eternalism is true, there is never a time at which the Joe not marrying Laura world is possible. But this isn't right. For what is possible is necessarily possible, consequently there is no time at which such a world becomes impossible, even after Joe's choice to marry Laura has been made. That seems to follow from the nature of possibility regardless of how time works

      Given that Joe in fact chooses to marry Laura, it is impossible that he not marry her. In other words, there is no possible world in which Joe both does and does not choose on Jan 1 2020 to marry Laura as this would be contradictory. But this does not mean it is impossible strictly speaking that Joe choose otherwise. So the eternalist should affirm:

      (1) Joe's choice to marry Laura is up to Joe and it is not determined by factors prior to the choice

      (2) There is a possible world in which Joe does not marry Laura.

      (3) This possible world is necessarily a possible world (by the necessity of possibility) and therefore it remains a possible world after 1/1/2020

      (4) Given that Joe does choose to marry Laura, it is impossible that he choose not marry her. In other words, there is no possible world in which Joe chooses to marry Laura on 1/1/2020 and he chooses not to do so.

      (5) Joe actually chooses to marry Laura on 1/1/2020

      And this brings me to the point about the present. In the present moment, I am freely typing. All should affirm (regardless of their views about time)

      (6) My choice to type is up to me and it is not determined by factors prior to the choice

      (7) There is a possible world where I do not freely type

      (8) This possible world is necessarily a possible world (by the necessity of possibility) and therefore it remains a possible world after 6/29/2017

      (9) Given that I do choose to type, it is impossible that I choose not to. In other words, there is no possible world in which I choose to both type and not type simultaneously.

      (10) I am actually typing in the present

      I think that (6)-(10) are perfectly compatible with libertarianism and I can consistently affirm (6)-(10) while also saying I am freely typing. And this is true for the presentist as well as the eternalist, since they agree about the ontological status of the present.

      Now, (6)-(10) are parallel with (1)-(5). Therefore, if we think (6)-(10) are compatible with libertarianism, then I would think we should affirm that (1)-(5) are as well. The basic idea is that for the eternalist, all points in time are ontologically on par. Therefore, if the presentist concedes that people act freely in the present, the eternalist thinks that the presentist should concede that eternalism does not undermine this point since the "future" according to eternalists is ontologically similar to the present in presentism.

      Tom

      Delete
  5. What is the Aristotelian-Thomistic position with regards to Cantor's discoveries about infinity?

    After all Cantor discovered that there is a listable infinity and a bigger one called unlistable.

    The infinity of natural numbers is listably infinite. But Cantor proved with various mathematical proofs that the infinity of the reals is bigger.

    And not just that, but that there are infinities bigger than the infinity of the reals too!!

    You can get those infinities by powersetting the infinity you have.

    For example, the powerset of the infinity of the natural numbers is the non-listable infinity of the reals. And you can do this with EVERY infinity.

    Cantor thus proved that there are an infinite amount of infinities, each bigger than the last.

    Because of this, there is no such thing as a set of all sets. The collection of all infinities (which is an infinitely large collection) is called a proper class.

    However, mathematicians have proved the existence of cardinal numbers bigger than even the infinity of all infinities.

    They are called inaccessible cardinals. And there are infinitely many degrees of inaccessibiliy.

    But inaccessibility is just the smallest of large cardinal properties, and there are biggers such as indescribable, Mahlo, Vopenka, supercompact etc.

    Now the biggest such property we know that is consistent is the rank-into-rank axioms, and the large cardinal properties may also go on forever.

    So what do Aristotelians think of these?

    BTW, I've read the google books version of An Aristotelian Realist Philosophy of Math, and the book doesn't really mention either Cantor' s infinities or the large cardinal properties in any good detail, so that is why I am asking.

    ReplyDelete
    Replies
    1. @JoeD:

      "Cantor thus proved that there are an infinite amount of infinities, each bigger than the last.

      Because of this, there is no such thing as a set of all sets."

      You are confusing two things. The fact that there is no set of all sets (in a formalized set theory like ZFC) is different, and easier (in the book where I first learned set theory it was left as an easy exercise to the reader before delving into the theory of ordinals and cardinals) from the fact that there is no set of all cardinals. Although there is a proof for the former which is essentially a formalization of the Burali-Forti paradox, there are easier ways to prove this.

      "However, mathematicians have proved the existence of cardinal numbers bigger than even the infinity of all infinities.

      They are called inaccessible cardinals."

      They are large cardinal *axioms* for a reason. What was proved is their relative consistency with ZFC (ok, Reinhardt cardinals and the like are out).

      "So what do Aristotelians think of these?"

      Why do you think that Aristoteleans, qua Aristoteleans, should have an opinion on this?

      Delete
    2. Well, some of them tend to view irrational numbers as beings of reason or ens ratio, which is a nominalist way of describing abstract mathematical entities.

      As for the large cardinal axioms, I knew they were axioms, I was sadly just sloppy with my commenting.

      But there is a book out there called Believing the Axioms by a philosopher who shows how reasonable these axioms actually are.

      Another thing that supports the axioms is the fact that large cardinal properties are surprisingly linear in their order

      Another thing about those infinities that interests me is the fact they tend to be quasi-mystical concepts which naturally bring about interest in people.

      Aquinas did mention in the Summa Contra Gentiles how the fact the intellect can always go further to infinity suggests the existence of something intelligibly infinite, which is God.

      And those new discoveries in math might just add to Aquinas's argument in some way.

      As for there being no set of all large cardinals, I didn't even know I was confusing with sets so thanks for the clarification.

      However, I think we could work with the assumption that there really is no set of all cardinals at least for the sake of the argument when it comes to A-T.

      Delete
    3. @JoeD:

      "But there is a book out there called Believing the Axioms by a philosopher who shows how reasonable these axioms actually are."

      It is a pair of papers by Penelope Maddy from the 1980's. Her philosophical position has changed since then.

      That axioms can be argued for is not in dispute -- at least not from me. What I asked was and I will quote myself "Why do you think that Aristoteleans, qua Aristoteleans, should have an opinion on this?" or why do you think large cardinal axioms are relevant to Aristoteleanism in any way.

      "As for there being no set of all large cardinals, I didn't even know I was confusing with sets so thanks for the clarification."

      I suppose I have to clarify my own clarification. What I responded to was,

      "Cantor thus proved that there are an infinite amount of infinities, each bigger than the last.

      Because of this, there is no such thing as a set of all sets."

      and what I objected to in this quote was the "because of this" part.

      "However, I think we could work with the assumption that there really is no set of all cardinals at least for the sake of the argument when it comes to A-T."

      I am confused by this. It is provably true (in ZFC say) that there is no set of all cardinals. Why would Aristoteleans need the assumption that "there really is no set of all cardinals"? For what? What do you think is at stake?

      Delete
    4. "But Cantor proved with various mathematical proofs that the infinity of the reals is bigger."

      Not really. He showed that we can give a sensible ordering to these infinities. But infinity is not a number, so I don't think the word "bigger" can even be applied.

      Delete
    5. @Gene Callahan,

      Well, at least the very fact that there are listable and non-listable implies that some infinities are greater than others in some sense, coming in at greater places in the hierarchy for example.

      After all, the reals are of a different kind than the natural numbers, and even contain them as well.

      Delete
    6. @grodrigues,

      The reason I am asking what Aristotelians think is, as I've already mentioned, because some Aristotelians tend to reject the existence of certain mathematical objects, such as pi. They view in a nominalistic fashion, as beings of reason or the mind. So that is what compels me to wonder what Aristotelians think of even more abstract notions such as large cardinal properties, especially if it turns out that there are infinitely many large cardinal properties.

      Some Aristotelians even question the whole terminology of set theory and it's foundations, and many Scholastics even attacked Cantor's idea of there being greater infinities as pantheistic and a challenge to the infinity of God when it was first proposed.

      Delete
    7. @JoeD:

      "The reason I am asking what Aristotelians think is, as I've already mentioned, because some Aristotelians tend to reject the existence of certain mathematical objects, such as pi. They view in a nominalistic fashion, as beings of reason or the mind. So that is what compels me to wonder what Aristotelians think of even more abstract notions such as large cardinal properties, especially if it turns out that there are infinitely many large cardinal properties."

      The problem with answering this is that the question is ambiguous and conflates different questions, some in metaphysics, some in the philosophy of mathematics, and different questions require different answers. I will say two things:

      First there are different senses in which one can reject this or that, or in which something is "real" (which, to quote one of my favourite authors, is one of those few words that without quotes is meaningless) or not. An Aristotelean could say that \pi is a being of reason and still have no problem in recognizing that it is an idea that can be put to good use. After all, the equations of GR, in invariant notation and geometric units, are: G = 8 \pi T. In other words, the questions have different answers depending on what you think mathematics is and what mathematicians are doing, but they do not necessarily impact what mathematicians are actually doing or interested in doing. But then what you should be asking is not what an Aristotelean thinks of large cardinal axioms, but what is the philosophy of mathematics that Aristoteleans would propose.

      To put the point in a slightly different fashion: even if you believe that large cardinal axioms are false, there are still good *mathematical* reasons to be interested in them.

      Second, I do not see how or why large cardinal axioms pose any *specific* problem that is not posed by, say, your average garden-variety infinities. Or even how or why large cardinal axioms are more abstract than the number \pi in any sense other than of being more removed from ordinary experience. But then so are lots and lots of other mathematical objects, even *finitary* ones. Just as an axiom is needed to make the jump from finite to infinite, axioms are needed to jump to higher orders of infinity, a fact alone that makes Set-theory experts interested in them. There are then all sorts of additional reasons for interest, from their intricate fine structure to the connection with issues like consistency strength and determinacy. Sometimes even mathematicians in general are, say algebraic geometers when dealing with not-necessarily small sites, because they at least have the virtue of reducing the pain and side-stepping technical issues (although to be fair, these issues *can* be solved, at least in the vast majority of cases). But these tend to be rare, because it is very rare that a mathematician will peek beyond V_{w + w}, and when he does it is in an "all or nothing" fashion like category theorists (e.g. the category of *all* this or that). But again, I do not see how or why an Aristotelean should be concerned with any of this.

      Delete
    8. But could an Aristotelian admit that Pi was not a being of reason but actually as real as the integers? Or all mathematical objects that have an objective rational structure (which includes large cardinals) for that matter in order to avoid what seems like cafeteria realism?

      In other words, could an Aristotelian propose a philosophy where almost all objects in math, if not all of them, are as real as the basic integers and are not to be interpreted nominalistically?

      Oh, and before I forget, you seem to be yourself very knowledgable when it comes to mathematics, so I want to ask you this because it is relevant to large cardinal properties:

      Does the hierarchy of large cardinal properties go on forever, or does it hit a finite ceiling due to the fact we have to watch out for inconsistency?

      The largest cardinal property we currently know not to be inconsistent with ZFC are the rank-into-rank axioms. And some think that there may only be one or two more of these properties before we hit the final amount of LCP's.

      Others however think that, even though there exist inconsistent cardinal properties like Reinhardt & Berkeley, the LCP hierarchy still goes on forever in strength without ever reaching inconsistency.

      But what actually is the truth of the matter?

      ---

      And about the greater infinities in set theory:

      Some Thomists criticise the idea of there being greater infinities because they define infinity as something boundless. Thus, they question the whole attempt to propose the existence of infinite sets because a set denotes something bounded.

      Is this a position a Thomist could reject in his philosophy of math and accept set theory?

      One more thing:

      When Cantor first proposed his idea of there being greater infinities, he and his ideas were heavily criticised by the Scholastics of the time for undermining the infinity of God and even were accused of pantheism, which Cantor vigorously denied (he was Lutheran after all and believed that God Himself revealed those infinities to him which would form the basis of set theory).

      Now, do you happen to know what the position of contemporary Scholastics on this issue? Have they changed their previous position which was a condemnation of Cantor's ideas?

      Delete
    9. @JoeD:

      "In other words, could an Aristotelian propose a philosophy where almost all objects in math, if not all of them, are as real as the basic integers and are not to be interpreted nominalistically?"

      My background is in mathematics, and in physics to a lesser extent; I do not have the knowledge or authority to speak for Aristoteleans (and it is my impression that when you say "Aristotelean" you really mean "Thomist"). With this caveat, I find your question puzzling. Aristoteleans are moderate realists; one way to characterize their difference with Platonist realists would be to say that they reject uninstantiated universals. So what do you mean by "all objects of math" are "as real as the basic integers"? I am assuming here that numbers are to be conceived as universals which is not all obvious (and probably wrong), but for my purposes here, it does not matter much.

      "Does the hierarchy of large cardinal properties go on forever, or does it hit a finite ceiling due to the fact we have to watch out for inconsistency?"

      I do not know what you mean by a "finite ceiling". I will interpret your question as asking if there is a top to the large cardinal axioms (LCA from now on) hierarchy. You should really ask the experts in set-theory not me, but my (naive and uninformed) expectation is no. One problem is that the question is really not well-defined, because is it not clear what counts as an LCA. The second problem is that large cardinals are not only about size (insert tasteless joke here) but embody deep combinatorial principles (which is one of the reasons why they are not at all obvious), have arithmetical consequences (consistency strength), consequences for the structure of the subsets of the real line (determinacy), etc.

      Kunen's inconsistency theorem (which has been generalized several times in several different directions) puts a hard cap on large cardinals, as they are usually (always??) defined or characterized as the critical points of elementary embeddings. On the other hand, here is an informal heuristic for why we would expect there to be no top. It is based on two observations. First, by Göedel's incompleteness theorems, if T is a consistent theory then T + Con(T) is a consistent extension of T. And you can iterate (e.g. T_1 = T + Con(T), then recursively T_i = Con(T_{i - 1}), then transfinite iteration in limit ordinals, etc.). The second observation is that the LCA's seem to be organized in a linear order, ordered by consistency strength; more, they seem to function as natural measures of consistency strength for consistent extensions of ZFC. This is a heuristic, not a theorem. Combine the two observations to get the result.

      There are a couple of additional reasons off the top of my head why this is not really a compelling argument, but this rapidly becomes too technical and I am not an expert, or even knowledgeable, so you will excuse me if I just keep my damn ignorant mouth shut.

      If you want to lean more about large cardinals start with Drake's "Set Theory - An Introduction to Large Cardinals". The book is an introduction and is by now outdated, but still pretty solid on (some of) the fundamental ideas. Then proceed to Kanamori's "The Higher Infinite".

      "Now, do you happen to know what the position of contemporary Scholastics on this issue? Have they changed their previous position which was a condemnation of Cantor's ideas?"

      I do not know the tenor of such criticisms and, if your description is accurate, I find them highly dubious. But at any rate, sorry, no I cannot help you.

      Delete
    10. When Cantor first proposed his idea of there being greater infinities, he and his ideas were heavily criticised by the Scholastics of the time for undermining the infinity of God and even were accused of pantheism, which Cantor vigorously denied

      This is somewhat mixed up. Early on, scholastics were the primary group interested in Cantor's ideas. The criticisms touching on pantheism were not due to the actual mathematical arguments, but originally when Cantor explained his views on the infinite, he often used expressions from Spinoza, and at a time when pantheism was becoming fashionable. He thus was actually saying things that sounded like pantheism. Cantor himself was well aware of the problem, which is why his most important discussion of religious matters is his correspondence with Cardinal Franzelin, in which he asked Franzelin's help in framing his philosophical position in a way that would not be objectionable. (Cantor modified his account to take into account all of Franzelin's arguments, and later often quoted Franzelin's approval of the modified framework, although it has been pointed out that later he will often treat Franzelin's arguments as correct while at the same time saying things not obviously consistent with them.) Thus the issue was entirely with Cantor's background metaphysical framework, and not with the mathematics as such.

      Delete
    11. @Brandon,

      That answers my questions about Cantor being criticised for pantheism, but what about Scholastics who criticised his idea of there being greater infinities as undermining the singular infinity of God?

      Delete
    12. Whose criticism in particular? There were several Scholastics, like Gutberlet, who argued the opposite.

      Delete
    13. Well, here:

      ''Some Christian theologians (particularly neo-Scholastics) saw Cantor's work as a challenge to the uniqueness of the absolute infinity in the nature of God[6] – on one occasion equating the theory of transfinite numbers with pantheism[7] – a proposition that Cantor vigorously rejected.''

      The number 6 links to a book by Dauben in 1977.

      Another quote:

      ''In particular, Neo-Thomist thinkers saw the existence of an actual infinity that consisted of something other than God as jeopardizing "God's exclusive claim to supreme infinity".[71] Cantor strongly believed that this view was a misinterpretation of infinity, and was convinced that set theory could help correct this mistake:[72]''

      The number 71 links to a book called The Catholics, the Cathars, and the Concept of Infinity in the Thirteenth Century from 1997 written by Anne A Davenport.

      ---

      Now that's about the opposition Cantor encountered.

      What I am also interested in, though, is what other consequences the higher infinities have.

      Cantor himself thought that, somehow, his discovery of higher infinities ran counter to materialism.

      Can this really be turned into an argument against materialism?

      He also thought that the existence of ever expanding infinities and concepts implied the existence of something that could not be increased in any way whatsoever, something Absolute, which is basically God.

      Now, modern mathematics has moved on from even Cantor's infinity of infinities. Large cardinal properties were discovered, in part, because mathematicians recognised that formal systems were always incomplete, and thus there would always be infinities bigger than anything that could be formalised in standard way.

      Thus, large cardinals properties were discovered. And even though the largest cardinal property we are aware of that isn't inconsistent are the rank-into-rank axioms, many mathematicians hypothesize that even the large cardinal properties are infinitely many.

      And that is not all. The universe of all sets and proper classes, called V, is basically a rebaptised version of Cantor's Absolute, yet even the universe of all mathematical notions can be expanded.

      See here: https://mathoverflow.net/questions/100981/ultrainfinitism-or-a-step-beyond-the-transfinite

      What this means is that any concept in math can be expanded on forever.And even the totality of all of THAT expansion can also be expanded. Now this itself might lead to the existence of Cantor's Absolute, as eternal expansion of all concepts and infinities must have a source somewhere, namely God.

      What do you think of this argument?

      This is, in fact, an argument Aquinas himself makes in favor of the existence of God in the Summa Contra Gentiles, by pointing out that the very fact that the human intellect can go on forever even past infinity points to something actually infinitely intelligible, which is God.

      Delete
    14. Notably, this doesn't actually answer the question. Which Scholastics in particular?

      Infinite intelligible arguments are interesting, and since they are important, it is good to see someone recognizing that they exist, but historically they have never been tied to any particular mathematical result, and require nothing more than that intellect can have the infinite as an object, which is a conclusion that can be drawn in a lot of different ways from a lot of different things in mathematics.

      Delete
    15. Well, in order to know what Scholastics are being refered to, one would have to read the books I mentioned where the claim was made that some Scholastics thought higher infinities might undermine God's infinity.

      But since you mention Gutberlet as a Scholastic who actually accepted Cantor's ideas and liked them, I think this can just be answered as nothing more than a disagreement between Scholastics.

      As for infinite intelligible arguments, the reason why I think people should talk about them again is because of the discoveries of mathematics.

      First came Cantor's proof that there an infinite amount of infinities, all of which are bigger than the last.

      Then we had large cardinal properties, which are so big that they are to infinity what infinity is to the number 0.

      The large cardinal properties may go on forever too, and if they don't, I'm pretty sure some people will be dissapointed.

      And in article I cited called Ultra Infinitism, you can even take the Universe of all Mathematical objects, which includes all large cardinal properties as well as everything else, and reach beyond it if you define the standard universe V as a hyperinfinite number.

      Here is a quote:

      ''Now, let us be brave and say: what about breaking through into the trans-transfinite?

      What about , for instance, starting from VV itself and state that its size is some hyperinfinte number, say ℵ0,1ℵ0,1 ?

      (SIDE NOTE ON NOTATION: The standard aleph series would now be ℵ0,0ℵ0,0 , ℵ1,0ℵ1,0, .... The second subindex controls the degree of hyperfiniteness, much like degrees of unsolvability. I could have put it on top, but then it would cause troubles with cardinal exponentiations ).

      Wait, I hear you say loud and clear. Are you crazy?

      Don't you know that there is NO SET XX such that X=VX=V? Don't you know that there is no max ordinal?

      Yes, ladies and gentlemen, I do know it. But I do reply: and so what? The objection is exactly the same as the one of the finitists vis-a'-vis ωω. Someone has broken through the finite, so why not the transfinite? There is no set, but who said that it must be a set?''

      He later goes on to define V as a M0 universe, and then describes a way to mathematically go beyond it into a much higher mathematical universe M1, and this obviously looks like it too could go on forever.

      This seems to prove that bigger infinities and even conceptual levels bigger than standard infinity are no problem, because the very reason people go beyond traditional infinity is because they find it interesting and can expand beyond infinity and infinite amount of times.

      So infinite intelligibility is preserved.

      One objection to Aquinas's argument from infinite intelligibility is that infinite objects exist in reality, so this does not lend support to the idea of God.

      The number Pi, for example, literally goes on forever in it's decimal expansion, and yet every number in that infinite series exists objectively.

      Another example could be the equations used to measure the Red Shift of black holes.

      The equations are notorious for describing the Red Shift actually increasing to infinity.

      So we have an infinite quality in the actual physical universe too.

      Another infinite object is the Hilbert space used in quantum physics.

      String theory also seems to require positing literally an infinite amount of dimensions in order to describe the vibration of the strings.

      Now string theory is in the realm of theoretical physics, and empirical evidence eludes us, but string theory so far is the best theory in theoretical physics that we have, outshining all of it's other competitors.

      So if string theory is empirically verified tomorrow, then this could be added to the objection.

      Delete
    16. @Brandon,

      Now, the new developments in mathematics could in fact offer a reply to this objection to Aquinas's argument

      Since the infinity described in the equations for black holes increases to the infinity of the natural numbers, it is not the biggest infinity.

      Even if it increased to the infinity of the reals, there would still be an INFINITE amount of ever increasing infinities.

      And then we have the large cardinals.

      And then we have the hyperinfinite numbers where we jump beyond the universe of mathematics itself.

      This ever increasing amount of infinite concepts could indeed point to God in the end, thus defeating the objection.

      Now, one response to the above defense could be that mathematics describes things that could exist in physical reality.

      One of the commenters from here, Mr Green, seems to hold this position.

      Thus, all mathematics describes the physical, thus blocking the argument from infinite intelligibility.

      And it would be interesting what Aristotelians would have to say about this since a lot of Aristotelians reject uninstantiated universals in math.

      Some, in fact, do not reject uninstantiated universals though, at least from what I've read of An Aristotelian Realist Philosophy of Math.

      Now, what interests me even more is Cantor's claim that his discovery of higher infinities goes against materialism in some sense.

      Do you think this is plausible yourself, or do you have problems with the claim that higher infinities pose a problem for materialism?

      Delete
  6. Also, an interesting thing to consider is the Argument from Logic for the existence of God.

    Now the laws of logic are absolute, they transcend physical reality as well as any and all dimensions of reality whether it be space, time, Hilbert or Bach spaces, String theory dimensions or whatever else.

    They also transcend other abstract concepts, immaterial realities like angels or spooky ectoplasm, and even absolute nothingness without concepts, abstract truths and possibility still follows logic.

    Therefore logic is absolute.

    Logic is also inherently intelligible; even brute facts follow logic as they cannot br both brute and non-brute.

    But according to classical theism, God is Being itself and Reason itself and therefore Logic itself.

    God is self-explaining, and Logic is self-explaining.

    So I think an argument for God could be made from sheer logic.

    Now one objection I can think of is that the laws of logic still follow the laws of logic, which is circular, but I dont really know if thats an objection or just trivia.

    Another objection could be made from Platonism where Logic is an abstract concept in a Platonic realm.

    However, Aristotelianism viws logic as just the nature of being itself.

    But it would be interesting to see if an argument from logic could be made without any metaphysics.

    ReplyDelete
    Replies
    1. Good point but personally I prefer to start from the argument of intelligibility. In this way one might avoid the counter of circularity because intelligibility is not a law although a law, logic itself, is intelligible but is not intelligibility itself.

      Delete
  7. Another interesting thing I've been thinking about is the infinity of certain patterns of thought.

    For example, I exist. And I know that I exist. And I know that I know that I exist. Abd so on ad infinitum.

    Now, what would be the Aristotelian explanation of that pattern of thought above that seems to go on forever.

    And can this be used to prove that the intellect is simple and non-composite because it grasps itself directly and without the need for infinitely long trains of thought?

    Also, an argument about the immateriality of the intellect I've been thinking about goes pike this:

    Nothing particular can grasp universals because it is particular.

    But anything physical is particular.

    Therefore the intellect is not physical.

    But the intellect is particular and not universal.

    Therefore something particular can grasp the universal.

    But why then say that it is non-physical rather than physical, since both are equally particular?

    ---

    So guys, can you help me out here?Which assumption is wrong? And if it is the first one that nothing particular can grasp the universal, then why say that it's an immaterial particular rather than physical?

    Oh and another thing about circularity:

    The human intellect grasps universal concepts.

    But what does concept mean? Is it also universal? What about the concept of universality? Isn't it circular for the intellect to grasp these?

    What about the concept of intellect? Can the intellect really grasp itself universally? Can the intellect, which is that by which we grasp universals, really grasp the universal concept of intellect, which is that by which we grasp universals?

    Isn't that also circular?

    What do you think?

    ReplyDelete
  8. any good books about final causality? i'm having some trouble understanding it.

    ReplyDelete
    Replies
    1. You could try "From Aristotle to Darwin and Back Again: A Journey in Final Causality, Species, and Evolution" by Etienne Gilson. I didn't find it the easiest read first time around but I think I'll get a lot more out of it now and I intend to reread it.

      Delete
    2. great, thanks!

      Delete
  9. I'm currently reading The Last Last Superstition and just finished learning the unmoved mover argument for God's existence, and have been left with two questions that I hope someone can help me answer.

    First, I am having trouble seeing how a purely actual being can be a coherent concept, given that there are potentials which cannot be simultaneously had. For example, if God is the actualization of all potentials, then why is He not a turtle? Presumably, being a turtle is a potential, but you can't both be a turtle and have all of the divine attributes of classical theism.

    Second, I am having trouble seeing why Dr Feser's argument that the first cause must be attributable to a single being. This given argument for a monotheistic conclusion goes like this: for there to be two wholly actual beings, there must be a feature to distinguish them. But there can't be differing features between two of the same of a wholly actual thing. For example, if the differentiating feature was that one was more powerful than the other, then that Being would be the wholly actual one, and the other would have the unrealized potential of being as powerful as the other.

    The problem I have run into with this argument, though, is that I can conceive of features these beings could have that would allow them to be different and yet, presumably, both wholly actual. For example, what if two different wholly actual beings had the differentiating feature of "being separate from eachother"? Is that just arguing in a circle, saying that they are different because they are different, or is this a real problem? Or what about the differentiating feature of having different names? Wouldn't that count? Hope someone will help me with this, but thanks to anyone in advance who reads this!

    ReplyDelete
    Replies
    1. @JoF:

      "For example, if God is the actualization of all potentials, then why is He not a turtle?"

      This is not what Prof. Feser says. What he says is that God is purely actual with no admixure of potentiality, not that he is the actualization of all potentialities -- which here you are using in the sense of possibilities which is also incorrect.

      Delete
    2. Wow, thank you for clearing up my misunderstanding--that helps a lot! Although, I'm not sure I see what you mean by saying that I have the wrong understanding of "potentiality", and am confusing it with possibility. Could you please explain what you mean by potentiality?

      Delete


    3. Try considering it this way: a plant can "sense" light, in that it can respond to light by turning toward it or leaning toward it. But it does not sense the source of light in any precise degree, nor does it sense all of the information the light carries, such as shapes, distances, etc. An animal can see the sun and see the shapes that the sun's light illuminates: the animal does not actualize the potential of "sensing" the sun in the precise manner in which the plant does, but what the animal does includes all that the plant does AND MORE. Yet the animal is not the plant, even though it actualizes all that the plant actualizes about sensing light. The animal does it in a higher mode of sensing, one that is more complete than that of the plant (one that includes awareness).

      But some animals see in black and white, they don't have color cones in their retinas, whereas others see in color. So the latter not only see the shapes and shadows which the simpler animals have, they also see colors from the light as well. The latter actualize all that the former animals actualize, and more. They do it in a higher mode of sensing than the B&W seeing animals, their eyes have the rods for seeing B&W and have cones for seeing color: their capacity for sight is in a higher mode.

      Now turn to humans. One man learns how to do the arithmetic of multiplying, and learns many individual tricks of doing it quickly, but understands nothing of the principle behind it all. Another man learns the tricks of doing division, but understands nothing of why it works. A third man studies math as a science, and learns the underlying principles that make multiplication and division work, though he does not bother to learn all of the separate tricks that the first two learned. The mathematician is able to do comprehend all that the other two do implicitly by understanding the principles by which they operate to multiply and divide. He doesn't need to follow their tricks.

      In an analogical way, we say of God that he has all of the positive actuality that a turtle has, and all that a lion has, and all that an eagle has, but he has them in a higher mode of being than as a turtle or a lion or an eagle. He knows all that a man knows, but he knows it in a higher mode of apprehension than through the bit-by-bit science of logic and reasoning, he knows it all more deeply through a single act of penetrating comprehension. Every mode of being that is good, he has in that higher mode that is also simple, so that he combines them all together.

      Delete
    4. @Jo F: a potentiality is a possible act, not yet actualized, that belongs to the nature of a being. In this sense, they could be seen as potential causes within the nature of being. So they are not "potential entities" but "potential acts". This means that in a being whose essence is a pure act, there are no potential acts and consequently there is no potentiality.

      Delete
    5. Thanks grodrigues, Theophilius and Tony for responding. Your comments have been very helpful.

      Delete
  10. Anyone have a response to the Maverick philosopher's argument here on how divine simplicity conflicts with God's knowledge of contingent truths:

    http://maverickphilosopher.typepad.com/maverick_philosopher/2017/04/divine-simplicity-is-god-identical-to-his-thoughts.html

    ReplyDelete
    Replies
    1. Anon 4:13

      I responded to BV's post here.

      Briefly, BV assumes that a contingent truth is always contingent because he buys into the possible worlds account of contingency. I think this is wrong, and that the real lesson of BV's post is that contingency is always contingency with respect to some later time. Taken in this way, God is understood to know all contingent truths when they happen, and in this sense the truth is a necessary truth, just as it is for us in the present moment when we know it.

      Delete
    2. This problem seems much discussed in the literature, some good stuff, some not so good stuff. I recommend reading the work of W Matthews Grant on this topic, he has an article in Faith & Philosophy from the 2012 issue dealing with your precise question. He and Mark Spencer also co-authored an article (which you can find online) which discusses different approaches on your question from a Thomistic perspective without endorsing a particular approach.

      My own thinking is this: it is incorrect to say that God's knowing P involves some intrinsic feature within the mind of God which corresponds to 'God knowing P' (be it an intrinsic property, mental state, event, etc.). Instead, I think we should affirm that 'God knowing that P' is in part constituted by P itself. Bernard Lonergan SJ held a view like this and one could argue that Aquinas supports such a position based on a certain reading of SCG book 2 questions 10-13 as well as De Potentia 8.

      If you are interested in a lengthier discussion, let me know, as I am very interested in this particular question and there is some interesting stuff out there to read on the matter.

      Tom

      Delete
  11. Dr Feser.... can I call Stardusty "gay" or is that too uncivil?

    (of course by "gay" I don't mean gay in the fun entertaining and fabulous way of Milo Yiannopoulos but gay in the sad creepy way of Perez Hilton)

    If not then I might still venture to defend the Papacy against that dude whatshisname who thinks Papal Infallibility and Papal Supremacy are wrong.

    Like Marty Feldman's Igor(pronounced Eye-Gor) I'm easy.

    https://www.youtube.com/watch?v=-XRXlFJzrRw

    ReplyDelete
  12. Are vaccines a counterexample to natural law theory?

    "Live Attenuated Vaccines: For these types of vaccines, a weaker, asymptotic form of the virus or bacteria is introduced into the body."

    From here: http://www.publichealth.org/public-awareness/understanding-vaccines/vaccines-work/

    ReplyDelete
    Replies
    1. Why would they be? Explain.

      Delete
    2. Sickness is a frustration of one or more bodily faculties. Getting a vaccine is in effect giving oneself a sickness.

      Delete
  13. Excellent. I do appreciate this outlet for discourse Prof Feser so graciously gives us.

    ReplyDelete
  14. For the sake of argument, let us assume that David Bentley Hart is right in his book, The Experience of God: Being, Consciousness, and Bliss that the intellect takes you to something like the Vedantic Brahman.

    What reason is there to believe in the Trinity, since the end culmination of the intellect is in something like the Vedantic Brahman? Shouldn't I just end there, and become a Vedantist?

    The only response that I can think of is that trust and loyalty surpass what the intellect can attain as far as the truth is concerned. Why should I trust and be loyal to the Christian Tradition more so than the Buddhist Tradition, Hindu Tradition, or Muslim Tradition?

    The next response could be that I am of European Slavic descent, and I should follow the Christian Tradition out of loyalty and not the Hindu, Buddhist, or Muslim. If that is the case, wouldn't a greater expression of loyalty be to follow the religion of my "pagan" ancestors?

    ReplyDelete
    Replies
    1. Well, for a start, we have next to no real knowledge of that pagan religion as it was actually practised, neopagans notwithstanding.

      Delete
    2. Another response could be: the better faith and reason stands together in a Tradition answering the fundamental questions, the stronger is the trust in that Tradition.

      Delete
    3. First of all, what do you mean by "faith". I was using the more "Roman" meaning of fides, which is where we get fidelity. So, it is more of a loyalty. What are the reasons for trust in a certain Tradition for you? Would an ethnic component be a factor?

      Take for example, what I said about David Bentley Hart's book where his version of the absolute is like the Vedantic Brahman. If this is the end culmination of reason, shouldn't the Hindu sit tight within his tradition? Do you believe the ex-Christian who becomes a Vedantist is morally blameworthy whereas the Hindu Indian is not, since you might see an element of betrayal or disloyalty in the ex-Christian?

      Delete
  15. This comment has been removed by the author.

    ReplyDelete
  16. Dear Doc, here's a potential topic of discussion:
    Where's your idea AT?
    I'm slowing crawling out of Plato's cave and meantime I'm trying to learn the traditional AT metaphysical system (buying all your books of course, doesn't this deserve a kind reply? :)
    Now you know better than me how "simple" is to answer the question above for a platonist. But for AT? The substantial form is in the matter, ok, but it seems that that form doesn't exist abstracted from the matter except in a mind. To avoid conceptualism, where's your idea AT?

    ReplyDelete
  17. How does one philosophically (not scientifically) respond to the objection that the entire universe or multiverse does not need a cause in esse to sustain it? Perhaps it's self existent or the underlying sea of cosmic energy, some form of prime matter, is the everlasting basis of reality. Furthermore, how does one go about proving the divine attributes after showing that there must be a cause in esse that sustains the natural realm in being?

    ReplyDelete
    Replies
    1. I would use the arguments of change and order or the "sum" of the two: the four causes. If we're going to face a philosophical inquiry then the argument of causes is unavoidable. Supporting the brute fact of a primordial energy or matter is not, imho, a valid philosophical counter since it doesn't account for the change and the order.

      Delete
    2. One could maintain, though, that postulating God, be it actus purus, Being, or a theistic personalist immaterial mind, is a brute fact itself, and that an appeal to primordial matter is just as justified as an appeal to God.

      Delete
    3. @Houdini,

      Postulating God is not a brute fact since the perfection of God, namely a omnipotent and omniscient being, is the ground for all perfections.

      Delete
    4. I don't see how being the ground for all perfections would somehow make God any less of a brute fact. I could say that the sea of primordial matter or energy underlying the universe has always existed and sustains our material universe in existence. Being the ground for reality in an extensive way doesn't automatically make you immune to the charge of being a brute fact. Even if we were to say that God was ultimately simple, there's still the question of why he's ultimately simple, and why he exists at all. Postulating the divine or the material as the basis for reality is equally absurd it seems, that is, we must ultimately come to a certain brute factness about the universe.

      Delete
    5. @Theophilius

      Good point, merely postulating matter or energy gives us a certain static basis of reality, but it doesn't explain change.

      Your second point,
      I think a Thomist, however, might take issue with you saying God is "a omnipotent and omniscient being," because God would just be a participant in Being and not Being itself, not the fundamental reality. So we would have to mount up and head further down to the causal series until we find the first mover, the ultimate explanation. That is, a theistic personalist account of God would need further explanation, and to say otherwise is to be guilty of a sort of brute fact mentality.

      @Houdini
      I'm sure many Thomists would see this as a straw man. Surely you can't think that Thomists are guilty of using God as a sort of arbitrary stopping point when explaining causal series of change or existence. The Thomist would hold that God isn't arbitrary, but rather he is logically necessary, a fundamental basis for all reality that we have to arrive at a posteriori by logical necessity. His simplicity, also, isn't arbitrary because he has to be simple or else his composition of parts would require an explanation, and would subsequently make him only a secondary member in some causal change.

      Delete
    6. @Houdini (and also @RomanJoe),

      For me a "brute fact" is a fact to be accepted without any reason for the existence of its being (@RomanJoe, I assume God as the only perfect being, pardon me for having not made it explicit). Now happens that the reason of the being of God is God himself, His being doesn't depend on any other being. In this sense postulating God is not, in my opinion, a brute fact: we have a reason for his being and it's really and ultimate reason :) Moreover since his being doesn't depend on any other being than this reason, literally: this being, grounds the being, the reason, of any perfection; so it's more an "intensive" reason than an "extensive" reason. To say the same for an energy it seems to me a lot more complicated since energy implies movement of some-thing so we have to give reason for the movement and for the some-thing, without these reasons I should say we're facing a brute fact namely a movement that move itself.

      Delete
    7. @Theophilius

      Okay, so why can't I just claim, like you claim about God, that primordial matter or energy (or whatever) has its reason for existence in itself?

      Delete
    8. @Theophilius
      Ah, thanks for clearing up your view. I'm still having trouble though. If God is just *a* type of being, as opposed to Being itself, then wouldn't he be composed of parts? That is, God would have separate but united attributes such as perfection, omnipotence, omniscience. But, as the Thomist would hold, we need an explanation for why these various parts are united, are in composition. To say that they just are, is to through PSR out the window and, contrary, to what we're trying to establish, that God has a sorta brute facticity (cool word, huh? I made it up) about him. We therefore have to appeal to something outside of the composition of parts which unites those parts. Eventually the regress has to stop and we will have to arrive at a purely simple reality, that is, Being itself, non-composed. This is what the scholastics call God.

      Delete
    9. This comment has been removed by the author.

      Delete
    10. @Houdini
      This seems hardly reasonable given our experience of material things. Everything material thing seems to rely on a cause outside of itself to sustain it in being at every moment. The chair's specific wooden structure, its property of rigidity, its molecular basis, its atomic basis, the subatomic levels. Why assume that energy or primordial matter is any different? We could also introduce the idea of matter and form into this. Matter is always of some form, energy is also of some form (be it a certain level, wave fluctuation). We would need an explanation why matter or energy has taken on a specific form. We can't just say "it just does have that form," lest we appeal to the same brute facticity we're trying to avoid.

      Delete
    11. To accept that substances are Hylemorphic is to accept that material things don't have the explanation for their existence in themselves. For prime matter is but pure potentiality and form but a mere abstraction. This is simply Aquinas' Third Way.

      Note, also, that to accept hylemorphism also entails the distinction between essence and existence in things, for the same reason the Third Way explains. But the distinction leads to the second way.

      Delete
    12. @Houdini

      "Okay, so why can't I just claim, like you claim about God, that primordial matter or energy (or whatever) has its reason for existence in itself?"

      Well that is just an assertion. Why actually think its true? The thomist isn't simply asserting it. The very idea of a being that is Pure Act/subsistent existence itself has it's reason for existence in itself.

      Also, i think the principle of proportionate causality has something to say here. If we just go by the PSR i agree that there is no strict incoherence in an atheist arguing for some element of the universe as the Neccessary being. But given the PPC that would seem to be false

      Delete
    13. Hi guys. I respond to this part of the thread here.

      Delete
    14. Nice post. So would you say that the universe obviously has an essence distinct from its existence--What it is doesn't explain that it is? Reason, however, leads us to believe that there must eventually be a close in this gap between essence and existence if anything at all is to exist.

      Delete
    15. Both the universe and God are per se necessary beings, but God has this perseity in the first sense that Aristotle lays out in Metaphysics V while the universe has it in the third sense.

      The criteria I use to distinguish the two in the post is that perseity in the first sense is such that the predicate is known immediately from the subject. We know the universe does not exist in this sense because we know what it is, but this does not suffice to tell us there is a universe. We don't know what God is, and to do so would require a different sort of knowledge than abstraction from sense and knowledge through likeness, as STA lays out in Prima pars Q. 12.

      Delete
    16. Ah yes good point. The universe does not inform its own existence.

      Delete
    17. @James Chastek,

      Can't the essence/existence distinction be derived from simply imagining that a thing were to stop existing and then that it were to pop into existence again?

      The very fact that you can even coherently imagine that something could pop out of existence and then pop back into it again should ALONE be a very strong indicator that the essence/existence distinction is correct.

      Delete
    18. @RomanJoe,

      Sure, a being whose essence is *being* (essence=existence) is not *a* type of being, is *the* Being. That means that the being of God is totally independent from any other being: pure act. Hence God is the absolute perfection, therefore *is* (not "has") omnipotent and omniscent since absolute perfection is being all the perfections at once since the lacking a perfection means dependency from a being. So, in my opinion no brute factility here (yep nice neologism), PSR is answered perfectly by the necessity of the pure act, the perfect being of God.

      Delete
  18. I've asked this to the author of the blog but never gotten an answer.

    At what point does the central insight of Berkeleyian idealism/immaterialism - that is, the inexistence of Matter and the sole existence of Mind - conflict with Aristotleanism?

    Obviously Airistotle et al use the term "matter". But it appears to me that this can easily be replaced by Berkeley's idea of "God's ideas".

    I simply cannot see a conflict here. If you blend the two you get a far more aesthetically pleasing worldview and it becomes much easier to understand ideas like: "God causes the world in space as well as through time".

    ReplyDelete
  19. I suppose that the conflict is exactly on the existence/not-existence of matter. For Aristotele it does exists, indeed. Maybe the can reach some form of agreement on the not-physical-existence of the prime matter.

    ReplyDelete
    Replies
    1. But why assume it? Aristotle's final appeal when determining the basic categories is to the senses. So is Berkeley's. Berkeley shows convincingly that matter is a redundant category as it cannot be given clear meaning.

      Berkeley argues that it is a shorter logical leap to assume that the constant ideas we experience through time in space are God's ideas because we have experience of Mind and can analogise to say that any ideas not controlled by a particular mind must be in God's Mind.

      This seems to me such a firmer basis for Aristotelianism rather than the somewhat fanciful and contentless term "matter".

      Delete
    2. Because... of the senses :) If there's no second matter (that is: prime matter with a substantial form) then what's the senses are for? My opinion is that Berkeley is right about the unclear meaning of matter but that's why is about prime matter, matter without a form. And it seems to me that B could match AT placing ideas in God's mind. I agree with you, Pure aristotelianism (as far as I know it) have this problem: where's your ideas at? :)

      Delete
    3. "If there's no second matter (that is: prime matter with a substantial form) then what's the senses are for?"

      They are the openings that we have to experience God's ideas. It's really simple. Much simpler and clearer than the classic Aristotelian base. And you can build the rest of the Aristotelian off of that rather than some contentless notion called "matter".

      Delete
    4. @TheIllusionist,

      One of the problems with idealism is that it messes with substantial change. Because if everything is made out of the same stuff, namely mind, then substantial change would not occur and everything would be accidental change.

      Yet another, more specifically theological problem, is that it messes with the Incarnation.

      Another problem is that Berkeleyan idealism is basically the philosophical prequel to post-modernism, insofar as the former denies the objective mind-independent reality of matter and the other goes one step further and denies that objective mind-independent truth doesn't exist either.

      Delete
    5. How on earth would everything be accidental change if the world that we commonly experience are God's ideas? God's ideas are not manipulable in way that our ideas are. There is a fundamental difference.

      As for the incarnation, I see no problem at all. Could you be clearer.

      That is a mischaracterisation of Berkeley's position. There is a mind-independent reality; it is called God's ideas. We cannot change them. They are subject to His laws. You're attacking Hume, not Berkeley*.

      *IMO Hume was a second rate philosopher who ruined Berkeley's extremely coherent system. Note that Hume generates absurdities, Berkeley studiously tries (and succeeds) in eliminating them.

      Delete
    6. @TheIllusionist,

      But senses as "openings we have to experience God's ideas" is exactly what implies the existence of a physical substance, namely second matter or "signed matter". Otherwise the argument instead of simpler become, in my opinion, quite unclear because we don't need matter, nor senses. If God want us to get directly His ideas (in fact that is the job done by our intellect even if with the sensible experience) what's the sense having the senses?

      Delete
    7. @TheIllusionist,

      If idealism is true, then the Incarnation is basically God becoming one of his ideas, which seems problematic.

      And the General Resurrection would also be problematic on idealism, unless you somehow make it completely compatible with Thomism.

      As for change, well, it depends on what idealism entails. If it entails that God's ideas are made out of one substance, then the problem I described appears. If not, then I guess it is avoided.

      Delete
    8. @Theo

      I think that you're confusing God's ideas with our ideas. The former are utterly different in nature.

      @JoeD

      I think you are too. God became flesh. Flesh is a God-idea. No problem.

      Delete
    9. @TheIllusionist,

      I certainly could confuse ideas but the point is that if I can confuse ideas it's because ideas are immaterial. But if matter exists only as idea then how can I avoid confusing my idea of matter from God's one? Moreover how can I get to know about the difference of my idea of matter from God's one? Again it seems to me that only the existence of a physical matter constitute the element, the idea if you want, that permits the knowledge of the distinction.

      Delete
    10. Berkeley's distinction is very, very simple.

      Picture a keyboard. Now picture a keyboard that turns into a sandwich. Okay. That's your idea. Note that it is under the control of your Will.

      Now look at the keyboard in front of you. Try to turn it into a sandwich. Doesn't work, right? That's because it's not your idea. It's God's idea. It is out of the control of our Will and only subordinated to His Will.

      Do you get me?

      Delete
    11. @TheIllusionist,

      Sure, I hope so :) Unfortunately for me the keyboard / sandwich distinction seems to be grounded in the assumption of the existence of physical matter. No question there: God could make a sandwich from a keyboard but this seems not His Will. Seems that it's not the way things work. He will not command to a stone to become bread but He will turn water into wine, or will multiply fishes. This means that even God is respectful of the nature of the substance that He create and He changes, in this case nutritive physical matter.

      Delete
    12. Yes. He could change it to a sandwich. And all he would have to do is think about it. He wouldn't have to send fairies down to do it.

      But here, try the negative case. Describe what "matter" is and how you know that it exists.

      Delete
    13. @TheIllusionist,

      I know He doesn't need fairies :) Let's go back to the question of argument. Matter, with form, is the co-principle of the material substances. Matter alone, prime matter, can't be described due to its lack of form: no form no description. How do I know that it exists then? Thanks to the material substances (I am following what I know of the Aristotelian system) that constitute the "raw data" for the senses from which my intellect abstract the matter and maintains the form, that's the idea. If the process of knowledge is without errors (and usually to the point of taking cognizance is without errors if the sensitive apparatus is working), I should have, more or less, the same idea of God.

      Delete
    14. @TheIllusionist,

      Well, if your view of idealism suggests a single metaphysical substrate as the fundamental reality, then several problems with A-T arise.

      First, form and matter are related to each other as being-in-act & being-in-potency. If there is no physical matter, then there is nothing individuate a particular and nothing to keep things as particulars under a certain substance. Thus, everything would actually be a subsistent substantial form like an angel. But substantial generation and corruption would be unexplained and indeed reduced to accidental change.

      Everything would thus be it's own genus.

      But a bigger problem is with your keyboard analogy. If everything were just ideas in God's mind ( mind being attributed to God in a classical theistic fashion, however it's supposed to work) then this seems to me to be too close to occasionalism for comfort.

      For my idea of a keybord turning into a sandwich doesn't have any causal power of it's own. All of the causal power stems from me willing that something happens, not from those ideas having any inherent causal power with which I merely concur with.

      And if we take this analogy as describing how God relates to the world (or rather, his ideas) we get what seems like basically an idealist version of the occasionalist picture of reality.

      Delete
    15. @Theo

      "How do I know that it exists then? Thanks to the material substances (I am following what I know of the Aristotelian system) that constitute the "raw data" for the senses from which my intellect abstract the matter and maintains the form, that's the idea."

      But how do you know that matter constitutes this raw data?

      Why not ideas in the Mind of God?

      Delete
    16. @Joe

      "If there is no physical matter, then there is nothing individuate a particular and nothing to keep things as particulars under a certain substance."

      Yes there is. God's ideas.

      "Thus, everything would actually be a subsistent substantial form like an angel."

      Not really. I'd think of them as ideas in the Mind of God that He does not actualise for human experience. Whereas he does actualise his idea of this cup in front of me for human experience.

      "...then this seems to me to be too close to occasionalism for comfort..."

      It seems to me that your argument here simply assumes that anything remotely occasionalist is Bad. But you have to make that case, not assume it.

      Yes, Berkeley was a reader of Malebranche. His idealism integrated elements of occasionalism and made them coherent by dumping others.

      Delete
    17. I suppose that the conflict is exactly on the existence/not-existence of matter.

      While it doesn't provide a definitive answer to the question, it's perhaps worth noting that Berkeley himself did not think this was a point of conflict between Aristotle and himself; he discusses it explicitly in Siris 311-318. (He raises the point that Aristotle recognizes two kinds of being, potential and actual, and this means that you can't necessarily assume that when Aristotle says matter exists he means it in the sense that someone like Locke would.)

      Delete
    18. I don't get how God's ideas are meant to give universals a place to instantiate themselves. A-T holds that universals and other eternal truths are grounded in God's intellect, so you would somehow have to hold that particulars are also somehow grounded in God's intellect. But that would pose the problem of how contingent particular entities could have such a nature that they co-exist as ideas with universal concepts & eternal truths. We would basically have a genus of things called ''ideas'' that somehow split into two different ''species'', which would be an absurd thing to suggest for immaterial ideas in God's own intellect.

      And how exactly does God actualise an idea? By combining essence and existence? Well what is the essence of an idea then? Another idea? Doesn't this mean that when ideas are not actual that they coexist with universals & eternal truths? Under A-T God actualises an ''idea'' by combining it's essence with existence, and form with matter at the same time. If there is no matter, then all that remains is form, and form is inert and needs matter to individuate itself. Otherwise what we would have is a metaphysical system where form is all there is, presumably combined with God's ''ideas'' to make it work with A-T, however that's possible, because otherwise we would have to reject the idea of form & matter needing eachother and by extension the essence-existence distintion because that distinction implies the relationship between form and matter.

      Basically, you would have to find a way to treat God's ideas in exactly the way A-T treats matter. Ideas would somehow have to be distinct from form, yet function in a way like Aristotelian prime matter.

      Now Bill Vallicella has pointed out some interesting similarities between consciousness and prime matter, but it's not really conclusive.

      A former commenter of this blog here called Scott Ryan was a Thomist who believed that Aristotelianism was compatible with panexperientalism and even with panpsychism, and even worked on a paper to show this. I don't know if he finished it though, and he did this when he was a Jewish thomist btw. He converted to Catholicism near the end of his life in late 2015/early 2016, so I don't know if he still held this position when he converted. He was also a panentheist of sorts before he converted too.

      So if you can completely synthesize A-T with Berkeleyan idealism, that would be great, but I don't think anyone counts on that.

      Occasionalism is still incompatible with A-T, and so is some other stuff about idealism insofar as it tends to a direction that many people, even actual idealists, have taken that states that there is no mind-independent reality and which is basically a rejection of what they call ''naive realism''.

      Delete
    19. TheIllusionistJune 28, 2017 at 2:00 AM
      I've asked this to the author of the blog but never gotten an answer.

      At what point does the central insight of Berkeleyian idealism/immaterialism - that is, the inexistence of Matter and the sole existence of Mind - conflict with Aristotleanism?

      Obviously Airistotle et al use the term "matter". But it appears to me that this can easily be replaced by Berkeley's idea of "God's ideas".

      I simply cannot see a conflict here.


      Are you guys aware that this looks just like "TheIllusionist" is the new, (sorry, not improved) Dianelos, rebranding himself? Observe the following quotes:

      Dianelos Georgoudis April 7, 2017 at 7:50 AM...
      Berkeley used a particular language, but his meaning is quite clear: All our being is experiencing. Thus our knowledge of concepts is also a kind of experience (one who knows something has an experience someone who doesn't hasn't). In particular: We have a given set of experiences (say the sensory experiences related to observing quantum phenomena), we have a set of experiences of thinking about them, and we experience forming at a new concept of abstraction (in this case quantum mechanics). So where exactly is Berkeley's “simple error”? Can anybody explain that to me?

      I have been an idealist long before I read Berkeley. My reasoning was really simple: As a matter of fact my own being is experiencing and nothing but experiencing.

      Dianelos Georgoudis June 13, 2017 at 1:11 AM
      “Hume concludes that we have no idea of mind-independent physical objects either, because he thinks we have no impression of such things. We have, for example, only this fleeting impression of a round whitish patch, that other fleeting impression of a round whitish patch, a fleeting impression of a “knocking” sound, etc., but no impression of any substance that underlies and ties together these different impressions.”

      I think Berkeley would agree with this text. Consider: If an existent (not just an object and not just a physical object) is truly mind-independent then whether it exists or not would make absolutely no difference to our experience of life.


      Dianelos GeorgoudisJune 26, 2017 at 9:02 AM
      @ Michael C,

      ?“On your view God is just giving us an impression of it now, and perhaps someone wrote about a God-given impression of it 2,000 years ago, but it has no existence in between.”

      Interestingly enough Berkeley himself answered that objection pointing out that on idealism existence is grounded in the mind of God, and that building down to the last atom of it is there in the mind of God, whether anybody experiences it or not.[1]...

      [1] Berkeley claimed that to be is to be perceived. Apparently somebody asked whether a tree was still there when nobody is looking. Here’s Berkeley’s playful answer:

      ”Dear Sir, your astonishment's odd
      I'm always about in the Quad
      And that's why the tree
      continues to be
      Since observed by, yours faithfully, God”

      Delete
    20. @Brandon

      I'm not surprised. I think that Berkeley's project was to reopen a space for metaphysical questions while tackling the issues raised by the early modern philosophers. And Thomists would do well to give this a hearing. Because right now, no one is listening to them.

      @JoeD

      a. God holds the universals. He allows us to experience them through the particulars He grants us access to. He is the seat of universals. We experience these second-hand through particular ideas - based on universals - that He grants us access to.

      b. Yes. The essence of the idea is God's direct knowledge of it. The existence, for Man, is the derivative knowledge that He grants us access to. God is still Pure Being. And we are just beings. He has access to the essences directly and then imbues them with existence for us so that we can experience these essences indirectly.

      I think the synthesis can be done. I've never seen a decent point of contradiction. But philosophy isn't my day job, unfortunately. Maybe I'll try when I retire. But I think Thomists should start a dialogue.

      Delete
    21. Watching:

      I take it you meant that the 'playful answer', ie. limerick, was by Ronald Knox (1888-1957).

      Delete
    22. @TheIllusionist

      I know that matter gives raw data because of the clear form/matter distinction. A chalk triangle and a stone triangle is a triangle. No problem in placing all the real ideas in God's mind, it's the creature's perception that is different.

      Delete
    23. @Theo

      I think I've been unclear in my question. How do you know that it is MATTER that gives raw data?

      I believe that this raw data are God's ideas and I have a pretty decent metaphysical argument to justify it. Do you have a metaphysical argument to justify matter's existence? (And of course referring to the form/matter distinction would be tautological here).

      Delete
    24. @TheIllusionist,

      It seems to me that most people have an innate understanding of matter/form.

      We may be able to build logical cases and discursively come to the conclusion that one or the other don't exist, but I don't think those ideas come to us naturally.

      Now it could very well be that what we think of as matter is just the way God allows us access to His ideas, but then we should wonder why God would create us with an innate sense that matter is real.

      Delete
    25. @bmiller

      That strikes me as a very poor argument. You may as well say that because we naturally assume that the sun revolves around the earth then it is true - and if it isn't that calls into questions God's intentions.

      Delete
    26. @TheIllusionist,

      "That strikes me as a very poor argument. You may as well say that because we naturally assume that the sun revolves around the earth then it is true - and if it isn't that calls into questions God's intentions.

      Well I didn't say it was a conclusive argument. Without getting into inertial reference frames and such I take your point as valid. Sometimes our common sense grasp of reality can be wrong as with optical illusions and the like.

      But it doesn't mean our common sense grasp of reality is always wrong either. I take it you agree that it is normal for humans to have an innate grasp of the matter/form dichotomy? Then I think it is legitimate to ask why.

      I've heard good metaphysical cases for immaterialism and as I mentioned, perhaps it is the case. But it seems one facet of the theory boils down to our innate sense of "material things" being real is just an illusion. For the purpose of us living our lives and interacting with the world, it seems we must treat "material things" as real whether we are being fooled or not.

      Delete
    27. @bmiller

      Is the matter/form dichotomy reasonable? It certainly is if you believe in matter. But if you don't then everything is just form - forms that are imparted to us in the form of God's ideas.

      The real question I think is where did this idea of matter come from. The more I think about it the more I think that it is not natural. Certainly the subject/object distinction is quite natural - and obviously true and useful. But why have we started imparting a metaphysical substance to objects that has no content whatsoever? The more I think about it the more that strikes me as unnatural and incorrect.

      I'm not an expert in the history of ancient thought. But I think that Aristotle might have taken this error on by way of trying to mediate between the Platonists and the materialists. Basically, he gave to much to the materialists. Then in modern times obviously materialism has become an obsession.

      I firmly believe that A-T metaphysics is perfectly compatible with the idealist interpretation and it actually renders it more powerful - especially in its critiques of materialism.

      Delete
    28. @TheIllusionist,

      The real question I think is where did this idea of matter come from. The more I think about it the more I think that it is not natural.

      I am just assuming the form/matter distinction is innate, but I could be wrong. Perhaps some of the other thread participants are aware of the historical discussions regarding immaterialism.

      Delete
    29. The real question I think is where did this idea of matter come from. The more I think about it the more I think that it is not natural.

      But have you actually tried to consider the point from the perspective of the Aristotelian rationale for form and matter? That's the tricky question. If you haven't, then it seems "not natural" because you haven't considered why it was put forward to begin with. Because he didn't do it because it "seemed natural".

      I can't do full justice to Aristotle's development in a comment, so you should go read the first three books of his Physics.
      He starts with change, which we observe. With many changes, there is a that with respect to which that is different from before, and there is a that which persists (not different), because we often say that "X undergoes a change", say, the grass blade grows longer. There is an aspect of difference, "longer" and an aspect of sameness, "grass blade". There is a subject which persists, and that of which the subject is now different. If the explanation for the quality in which the subject is now different is "form", the subject must be explained in terms of "that which is capable of receiving this form or that form."

      But the same argument applies to change that regards not just qualitative change with the same entity being there before and after the event, like the blade of grass. We say of substantial change (i.e. change of an entity itself) that there is something that is different (what was rabbit is now wolf, because the wolf ate it), but also something that persisted or carried through the change: the wolf gained something by eating the rabbit, and it did not gain the form "rabbitness". The explanation, then, is that what persists "underneath" or besides the change of form is "matter". The matter of the rabbit became matter of the wolf. Which means that matter is capable of being the subject not only of this or that qualitative form, but of this or that substantial form. To Aristotle, that's what matter is: the subject of substantial change. You can't get rid of matter unless you either want to deny substantial change, or you want to deny there is something that persists through it.

      It is indeed possible to say that it is only an appearance that the wolf gains by eating the rabbit, and that in God's mind the form of the wolf is granted the appearance of a larger form when it undergoes the appearance of eating the appearance of the rabbit. (If the form is not inhering in matter, "appearance" is all the form gets you, that's all it does is appear). But saying it does not make it make sense: you end up merely positing that God makes all the appearances come out as if the form and matter were both real, without any good reason to deny the reality of them to begin with.

      Delete
  20. Does anybody have any thoughts or article / book recommendations on reconciling an acceptance of PSR with libertarian free will? I'm not particularly committed to libertarian free will but I've seen this charge made frequently.

    From my understanding PSR generally requires that contingent entities, events, etc. have some type of extrinsic explanation. And on the libertarian view of free will our choices are contingent, yet they're often viewed as being self-explanatory, at least in the sense that they're not determined by prior causes.

    ReplyDelete
    Replies
    1. How about a "simpler" re-formulation of PSR in a "causal-sense": every change require a cause? Action means change, we are the cause of our action so we are a cause-of-change and the action is take on reason determined by us. Arbitrarily or reasonably? Whatever the answer it follows PSR and doesn't undermine free will.

      More (and more professional) here:

      https://www.amazon.it/Principle-Sufficient-Reason-Reassessment-Philosophy-ebook/dp/B000T08VKM/ref=asap_bc?ie=UTF8

      Delete
    2. Hey--what do you know, I've been reading this essay (http://alexanderpruss.com/papers/LCA.html) by Dr. Pruss today.

      Delete
  21. 1. What is the best possible way to understand evolutionary theory in terms of AT metaphysics (books on the subject appreciated).
    2. Is there a standard book on AT philosophy of science.
    3. It seems obvious to me that, properly understood, natural law theory is more compatible with evolutionary theory than it's opposed to it, since a large part of our natures' ends are going to be related to and illuminated by a focus on survival.
    4. Do you have a final assessment of Alistair McIntyre's work?
    5. How is God's being pure act to be reconciled with Christ's being spatiotemporally located? It's not obvious that simply claiming that he has both a divine and a human nature is sufficient to make the idea intelligible without resort to some herrisy or another.

    ReplyDelete
    Replies
    1. Dear iwpoe: James Brent has a relatively new book out called "Thomistic Evolution: A Catholic Approach To Understanding Evolution In The Light Of Faith." I think many chapters of the book are available for free online, but you might as well get the whole thing together.

      Delete
    2. For a verdict on McIntyre, I suggest Emile Perreau-Saussine's 'intellectual biography' of him, and the preface by Pierre Manent.

      Delete
    3. Iwpoe,

      1.) Volume One of the Complete Works of Charles DeKoninck. Short answer: matter is part of the essence of animals and so evolution of essence is both possible and likely.

      2.) The fullest statement on this is Thomism and Mathematical Physics by Bernard Mullahy. Strong suggestion: download it and have Kinko's print it and bind it for you for $60. Then take six months to read it.
      3.) -
      4.) -
      5.) "Reconciled" is probably not the best word. All classical accounts of what Christ is presuppose that God is simple or incorruptible or non-spacial, so all classic christology is already a rapprochement between these two natures. On my reading of the Tome of Leo, for example, Christ has those double natures from his two births, from God and from the Virgin.

      Delete
  22. Does the Catholic Church have a definite claim about temporal punishment for heresy, or has this changed dramatically over time?

    ReplyDelete
    Replies
    1. Billy, the Church had, at various times, pretty strenuous claims that it is licit to punish heretics under the civil authority.

      The underlying requirement for this is two-fold: (a) a "heretic" is someone who did have the true faith, and the repudiated it. In Catholicism, though, having the true faith (at least as an adult) involves not only believing certain things, but promising to do certain things. Breaking your promises is something punishable in the temporal order, even if it is due to a change in your "sincerest beliefs".

      (b) A condition that the state is itself a Catholic state. This does not require that the state either be a theocracy or that it "enforce belief", all it requires is that the government have enough of a formal recognition of Catholicism that it willingly asserts such temporal assistance to the well-ordered Catholic state as to uphold certain religiously-based but temporally effective states and needs. For example, this can be as simple as officially recognizing as marriage whatever the Church holds as true marriage, and officially holding as null those "marriages" the Church declares null. And (in this current context) requiring of those who have failed their definite religious obligation to keep their consciences free of heresy, to not publicly teach their heresy, as was implicitly promised in their receiving sacraments, such as confirmation.

      Since Vatican II and "Dignitatis Humanae", many people have erroneously thought that the Church no longer teaches that a state can be "a Catholic state". However, the Church did seemingly turn its back on state punishment specifically for heresy itself (rather than for peripherals usually coincident with heresy). My sense from DH is that it can still be read to permit state punishment, not for heretical belief but for other overt acts that are frequently found with heresy, such as using unjust methods to try to convince others of your heresy (i.e. crimes against truth). Most heretics would not bother to draw the distinction and claim that they are being persecuted for heresy, but the distinction is there.

      Delete
  23. Has anyone read or is planing to read Sean Carrol's new book on humanism? It has an awesome cover that may factor into me buying it.

    I hope Dr Feser will write on these atheist public figures. Sean Carroll so far seems to me a much less philosophically naive atheist when compared to the new atheists.

    He had a fascinating debate with William Lane Craig and is one of the foremost cosmologists at the California Institue of Technology. I'm curious to hear Dr Feser's response to his arguments against the existence of a soul and God. (Why does brain damage affect mental function, do you get a copy of all your memories translated over to your soul when you leave your brain behind, does the soul develop with the human body from conception and how does it get there, from the parents?). That last point is front Tim O'Connor, though.

    ReplyDelete
    Replies
    1. Nothing quite like physicists / public communicators of science pontificating on issues outside of their domain.

      Humanism is, to me, illusory. Nothing but a self-aggrandizing view contradictory with the humanist's underlying view of reality.

      As regards brain damage affecting mental function - this might be a problem for Cartesian Dualism, but it is no problem for hylomorphic dualism. If I may quote from Dr Feser's book Philosophy of Mind:"...on the Thomistic view, the soul is (almost) as close the the body as the form of the chair is to the matter of the chair. Just as the form of a chair cannot function apart from the chair's matter, neither can the soul, for the most part anyway, function apart from the matter of the brain and body."

      In short, Hylomorphic dualism holds that the intellect is a wholly immaterial substance, but sense and perception are purely material and could not function or exist apart from the body.

      As for the Carroll and the existence of God: Given his ignorance of classical theistic argumentation, he is mainly familiar with the modern approach to natural theology that invokes scientific claims. Thus he seems to assume theism is a question of whether the universe began to exist / explaining complex phenomena.

      It seems to me that the Existential Argument of Aquinas (found in De Ente Et Essentia), for instance, utterly trivializes all the common objections to theism people like Carroll would raise.

      Make no mistake, Dr Carroll is still very much so naive. Less naive than others, alright; but still very naive.

      I would recommend Feser's books 'Philosophy of Mind' and 'Aquinas' to learn more about the Thomistic/Hylomorphic analysis of the soul/intellect. Also, David Oderberg's paper on Hylemorphic Dualism: https://drive.google.com/file/d/0B7SKlRTfkUieRWJGMzRadUVaXzg/view

      Delete
    2. Thanks for the book recommendations--I've dedicated this summer to reading up on anything relevant to the debate on atheism and Christianity's rational justifiability, so they will be helpful.

      Delete
  24. Two, if I may. (The second is really an extension of the first. And any knowledgable person may also comment.)

    (1) As an economics major interested in the intersection of faith and reason, science and philosophy, etc., are there any works you could recommend that touch on economic subjects (e.g. money, finance, interest)? Dempsey's Interest and Usury, the second volume of Cronin's Science of Ethics, and the multiple volumes on the Hispanic Scholastics are already on my to-read and currently reading lists.

    (2) In relation of faith to the social sciences, any thoughts on DeHart's article "Madisonian Thomism" over at Public Discourse? http://www.thepublicdiscourse.com/2017/01/18427/

    ReplyDelete
    Replies
    1. Heinrich Pesch springs to mind, as well as Wilhelm Ropke.

      Delete
    2. Also on my to-read list. (Perhaps I've found all such books, and just need to dedicate time to reading them now?)

      Delete
    3. Anything by Thomas Sowell. He brings up the moral arguments against various political philosophies, which tend to be rooted in a Christian worldview. Not sure how to make any recommendations relevant to Thomism, though, but I suppose you could argue from Thomistic morals to certain conclusions in political philosophy.

      Delete
    4. The Distributists seem to often draw from Thomism and Catholic social teaching. The Distributist Review online is a little quiet these days but is still going.

      E.F. Schumacher wished to call his Buddhist Economics, Catholic Economics, but thought there would be too much prejudice against it. He often drew from Thomism, as his Guide for the Perplexed makes clear.

      Delete
    5. @JLo: Sometimes I wonder just how much Sowell influenced my decision to study economics.

      @Jeremy: I was originally put off by them, but have regained interest since reading John Mueller's Redeeming Economics (ISI Books, 2014). Don't know why TDR insists on pushing the worst of the heterodox schools of economic thought.

      Will have to take a look at Schumacher.

      My main interest in Ed's suggestion because of some comments he made in his New Oxford Review article on Wood's The Church and the Market (http://www.newoxfordreview.org/briefly.jsp?did=0705-briefly). Would he simply recommend Cronin, or are there other works that contributed to his judgement that, concerning the relationship between economics and moral theology, "it is clear that Catholic critics of capitalism have not made this relationship any clearer than Woods has"?

      Then again, maybe it's best that there's no clear answer on such an abstract question, as I have read more in order to find the answer, the situation being otherwise.

      Delete
    6. Just realized my initial question wasn't *that* abstract, but the question regarding the relation between (social) science and faith is.

      Delete
    7. It has been, unfortunately, too long since I thought deeply about economics. I must admit, though, I have always leaned to the heterodox side in economics. Piero Sraffa, Joan Robinson, and the post-marginalist Keynes are, in my opinion, some of the most important economists of the twentieth century, even if I don't follow all their positive vision for the economy (I'm no social democrat). I'm also a fan of some less renowned heterodox economic thinkers, like Silvio Gesell, Ralph Borsodi, and Kevin Carson.

      Delete
    8. @Jeremy Taylor: Thanks for the suggestions. Just downloaded a book of Sraffa.

      Which writings of Keynes' are from his post-marginalist years?

      Delete
    9. Never mind. Just never heard "post-marginalist" in reference to the post-Keynesian school.

      Delete
    10. I mean post here not in the sense that it is used in the epithets post-structuralism or post-keynesianism itself, for example, but more or less the abandoning of much of the marginalist legacy: non-margibalist From 1937, with summary The General Theory of Employment, he seemed to increasingly abandon marginalism for a more dynamic and realist understanding of the economy. The post-keynesians very much represent Keynes's true legacy in this regard.

      Ever read Steve Keen's Debunking Economics?

      Delete
    11. No, but one of my professors has a similar book (What You Don't Learn in Economics 101, I think). My professor was very knowledgable about Minsky and his financial instability hypothesis, though.

      I tend to identify Keynesians (esp. Minsky-ites) with a philosophical nihilism that views creation as inherently evil, though I don't know if that's a correct assessment.

      Delete
    12. Post-keynesianism, when legitimate, is just analysis of the economy. It doesn't really have a vision for the whole of life or even how we organise society. Individual post-keynesians tend to be social democrats and soft leftists.

      Neither the post-keynesians nor neoclassical nor Austrians have a vision for man and society rooted in traditional metaphysics or Christian principles. For that one must look to people like the Distributists or Wilhelm Ropke. The post-keynesians (the neoclassicals, and the rest) are good for economic analysis only.

      Delete
    13. Borrowed's Polanyi's Full Employment and Free Trade through my university's library. Found the first line of the Introduction somewhat ironic in light of our discussion: "The present book, written by an eminent chemist, containst an important and interesting message. The author is a convinced Keynesian and ... a staunch individualist" (ix).

      There's also a book I think you might find interesting, John D. Mueller's Redeeming Economics: Rediscovering the Missing Element (ISI Books, 2010; updated paperback 2014). It has some interesting insights, and highlights Pesch and Ropke as important Christian economists.

      Delete
  25. Now, a general question -

    From a recent post by J. Budshizevski:"However, the second difficulty in your letter is more fundamental. I embrace the classical approach to epistemology which sets things before knowledge. You embrace the modern approach to epistemology which sets knowledge before things. This has been a dead end. Even the skeptic has to assume that something is true; otherwise he has no way to decide what to do and how to live – the springs of action lose their springiness. He cannot even justify his skepticism. One must first try to know something, then go ahead and criticize the power of knowing. We find out the weaknesses of the reasoning power only in the act of using it."

    Any articles that flesh out this distinction and the pre-modern "critique" of modern epistemology?

    ReplyDelete
  26. Can any Thomists comment on Bill Valicella's blog:

    http://maverickphilosopher.typepad.com/maverick_philosopher/2012/01/creation-ex-nihilo-or-ex-deo-am-i-a-panentheist.html

    Which basically proposes panentheism (with the extra 'EN') as a solution to the seeming contradiction of Creatio Ex Nihilo and Ex Nihilo Nihil Fit? Bill says that this can only be solved if by Creatio Ex Nihilo we really mean Creatio Ex Deo. That is, when we say God creates from nothing, we really mean God creates from nothing distinct from himself.

    Red flags and alarms are going off, but I have difficulty reconciling the two phrases myself.

    ReplyDelete
  27. Scott, I would say that Bill is just confusing two senses of "from" in "nothing comes from nothing" and "God creates from nothing." Furthermore, the first sentence really should not be stated in the form of an affirmation (that's the form it has) but in the form of a negation: it is not true that _something_ comes from nothing. Or: Something does not come from nothing (SDNN). Then the CEN statement that "something comes from God" is simply not in real tension with the SDNN.

    When God creates, the thing created is neither part of God nor "independent" of God in the sense that God does not have to maintain its existence. But this is not a dilemma or a paradox properly: created things depend on God continuing to act upon it at every moment and at every aspect of its being, but its being is not in any sense "from" nothing as from nothing as a material cause (or any other sort of cause) nor "from" God as from a material cause. IN CEN tt is "from" nothing in a negatory sense that it is not from any OTHER cause but God.

    So the whole issue can be resolved by saying them more clearly:

    Something does not come from nothing as from a cause.

    In creation, something comes from God without any other cause.

    There is no tension here.

    ReplyDelete
    Replies
    1. Thank you. That helps a lot. I was beginning to think along those lines when I read your reply. I also remembered that panENtheism as well as pantheism both necessarily violate divine simplicity. Obviously all essences pre-"exist" in the divine mind. However, that does not mean that their existence itself is contained within the divine substance. Rather their existence is efficiently caused by the Actus Purus (efficient causation implies a complete distinction, otherwise it would be formal or material causation). However, the essence must always remain in the divine intellect and be conserved in the finite being by the Actus Purus/Subsistent Being. Hence the immanence of god while maintaining the true transcendence of God. The important part of the "Creatio Ex Nihilo" is the "Creatio" part. Thus, Ex Nihilo Nihil Fit means nothing can come from absolutely no cause whatsoever, whereas Creatio Ex Nihilo means that God can create without any pre-existing substance. However, it does allow him to efficiently cause substances. I could see how someone could reword this as Creatio Ex Deo; however, the important distinction is that creatures (including all of reality apart from God) do not make up any part, no matter how finite of even an infinite God.

      Delete
  28. Having read The Last Superstition I don't think Feser sufficiently addressed the matter of why the final cause of humans is dictated by biology in the same way as is true of non-humans. The very fact that we are intellectual beings suggests the possibility that our final cause might issue in a transcendence or even a defiance of biology in order to attain a higher level of flourishing. I'm a member in good standing of the despised tribe of secularists, but I'm also dubious of naturalism/materialism. I haven't delved into Aristotle, much less Thomism, in decades, but now I'm intrigued by some aspects of the Aristotelian vision, particularly its avoidance of naturalism. However, as a secularist I'm also repelled by many features of the A-T worldview espoused so polemically by Feser, especially its morality.

    Any thoughts on the possibility of a secular interpretation of human final causality?

    Sogn

    ReplyDelete
    Replies
    1. Ultimately, I don't think a secular interpretation could work. Considering that we could not exist even for a moment if God weren't constantly sustaining us, it follows logically that God is our highest end.

      Delete
    2. Assuming, for argument's sake, the A-T arguments as deployed by Feser for God's existence are sound, it wouldn't necessarily affect my fundamental secular moral principles. My question wasn't meant to question God's existence, nor that God is our ultimate end, but rather to wonder why our, shall we say, penultimate earthly ends must be dictated by biological factors as though we are non-human animals rather than rational animals. Of course that is implicitly to suggest that God's own moral nature would be other than expounded by Feser, i.e. not Catholic but rather more in line with some secular (or even liberal Christian) values. To take one very culturally salient example, do Aristotelian arguments for an unmoved mover, etc, really entail that the interests of a woman should be subordinated to the interests of a zygote?

      Delete
    3. Any thoughts on the possibility of a secular interpretation of human final causality?

      I do think its possible considering that Feser himself says that Final causality by itself don't get one to God of classical theism,that Aristotle himself didn't think that God was needed to explain teleology in nature,
      And its certainly seems possible to accept some more liberal interpretation most Aristotelian Principles considering that many Neo-Aristotelians in contemporary philosophy are Non theists ..

      Delete
    4. Mill-Scout questions whether Aristotelian arguments "entail that the interests of a woman should be subordinated to the interests of a zygote?" I am assuming that the reference is to abortion. But the statement assumes that a woman's total interests (her good) are enhanced by killing the zygote. Her material situation may well be enhanced, but what is good has to be determined by looking at spiritual welfare as well, and by taking life's purpose into account.

      Delete
    5. but rather to wonder why our, shall we say, penultimate earthly ends must be dictated by biological factors as though we are non-human animals rather than rational animals.

      Mill-Scout, I think you may be taking Feser's account of biological influence in a way he didn't intend, nor Aristotle.

      Under standard the A-T approach, it isn't biology as such that "determines" the ends for a being, and certainly not biology conceived as the modernist physico-biology. The ends are determined, rather by the nature of the being, such as an animal or plant. The biology gives us a great deal of insight into that nature, but properly speaking the nature is more due to the "form" than the "matter", so in the A-T model you would always have to look beyond the merely material anyway.

      This is especially true of man, who while he is in the genus 'animal' is also rational, which is distinctly a spiritual dimension not found in the other animals. This is important here because the spiritual nature of man actually revises (I would say, revolutionizes) the whole of man, including his more animal-like faculties, to be reason-centered ultimately.

      For example, in an animal the pleasures entailed in the taste of the right foods are wrapped up in the teleology of eating what is nutritious for the animal, i.e. serves it to sustain its life. In man, it does that too, but it ALSO serves to give us knowledge about the world, which serves our reason, and helps enable us to understand higher things as well. Similarly, sexual pleasures, in man, not only serve for reproduction of the species, but also for the self-giving love of others in the marital friendship and the family, which is a good of the rational order, not just the good of the species. Properly speaking, the 'animal' faculties, in man, are really analogous to the similarly-named faculties found in animals, because in man they are integrated into a system (by man's rational nature) that makes those faculties formally serve rational goods.

      So, the biology needs to be recognized primarily as clear signs and indicators of the nature of the being, but it is the nature that is determinative.

      Delete
    6. Or, to complete the point: even our earthly ends are geared to our rational goods: to understanding the world, to friendship and love. The "good life" (in this world) is, essentially, a "life of reason", which Aristotle analyzed and identified as the life lived according to virtue because it is that life that integrates all the parts of man's needs and desires into a coherent whole with the lower serving the higher.

      Delete
    7. Michael wrote: Her material situation may well be enhanced [by killing the zygote], but what is good has to be determined by looking at spiritual welfare as well, and by taking life's purpose into account.

      The implication seems to be that by taking into account a certain view of the afterlife we could see that women would ultimately be better off by submitting even to unwanted pregnancies. On Feser's theology women's spiritual welfare is jeopardized merely by having sex without intending pregnancy (even if raped!). I don't know if Michael has such extremism in mind. All I can say is that this boils down to the ethical aspects of God. Personally, I don't deem such a being as morally good, and thus wouldn't even call it God, much less worship it. But the logic of the situation as posed by Michael is sound in the sense that it's conceivable that women could be damned for defying the purpose of a higher power with a women are reproductive vessels ethic.

      Delete
    8. Anonymous wrote: it certainly seems possible to accept some more liberal interpretation most Aristotelian Principles considering that many Neo-Aristotelians in contemporary philosophy are Non theists

      That's about what I was thinking. I see no reason, pace Feser, to proceed from Aristotle to Aquinas. I'm personally intrigued by the possibility of non-naturalism without theism, or at least without Catholic/conservative theism. Not that my bias has any dispositive weight whatsoever. ;-)

      Delete
    9. There are a couple other replies I still hope to make, to Tom & Tony.

      Delete
  29. Is all being causally efficacious? It seems like some being, such as potency (which is neither nothing or actual being) isn't causally efficacious.

    ReplyDelete
    Replies
    1. Potency itself would not be causally efficacious; it would have to be moved from potency to actuality by something already possessing actuality.

      Delete
    2. Okay, so not all being is causally efficacious? But we would also have to admit, I guess, that pure potency, independent of act, doesn't actually exist.

      Delete
    3. Right. Pure potency isn't being. For a being that has some potency, it is not being in the respect of which it is in potential. So it's potential is not "being" either.

      Delete
    4. I find this rather confusing. If we are going to admit that potency isn't in being then we'll have to admit that being can come from non being. So when I spill a glass of water, the potential for the water to be spilled is actualized. But if we say that a potential isn't in being then we would have to say the non being of the water was turned into being via the actual motion of my hand. But this is absurd. I think, then, that potency is a type of being just as act is a type of being. And both potency and act are distinct beings that, as the first thesis says, divides all being.

      Delete
    5. Only being-in-act is causally efficacious.

      Pure potency and potency are not the same.

      Potency is, properly speaking, being-in-potency. Being-in-potency (potentiality) is always rooted in some being-in-act (actuality).

      Pure potency is not the same as being-in-potency. Pure potency has no being at all. (Just as prime matter, which is matter without form, is pure potency and thus has no reality).

      Delete
  30. Since it is open I'll just say that I'm loving the reference to "The Conversation" one of my all time favorite movies.

    ReplyDelete
  31. Who killed Sister Cathy?

    ReplyDelete
  32. What would you say to someone who was choosing between Roman Catholicism and Eastern Orthodoxy? The more I think about it, the more it seems to me that the differences are largely political. Or am I mistaken?

    ReplyDelete
    Replies
    1. Many of the differences are political. But some are not. Obviously, the filioque is one example. When I was wondering about this, two issues I looked at were:

      1. The arguments against Papal infallibility from a very few popes (Liberius, Honorius; now Francis may well join the list), vs

      2. The problem of conciliar infallibility posed by the latrocinium of Ephesus.

      I ended up concluding that #2 was the bigger problem.

      One political difference is potentially big. It was only in the Catholic West - so far as I can tell - that actual independence of church and state arose. In fact, much of the "statist" rhetoric directed at Catholicism seems really to amount to indignation at the fact that the Church will not accept that it is under the state (as both Protestant and Orthodox churches are). Whether that should count here is debatable; I think it does in a secondary sense.

      Delete
  33. I have a request: What are the pertinent texts, Scriptural and magisterial, which undergird the Sede Vacantist claim that there is no present pope. And, what is the correct way to understand those texts?

    I gather that there are texts that say, roughly, that a 'pope' who is a heretic is not actually pope. However, "is a heretic" has been the subject of a great many disputed and debated and unsettled arguments, which is why (among other things) there is a Church here to declare explicitly and personally that "X is a heretic". So anything SHORT of that explicit declaration by the Church, if it satisfied sufficient conditions that the result "therefore X is not pope" would fall out automatically from the starting point stated above, would necessarily create a great deal of new doubt and disorder among good and faithful Catholics. Enough doubt and disorder, seemingly, that these would be as damaging to the Church as the heretic pope would be, if not more. So, for that reason alone, I doubt that can be the right interpretation of the texts.

    ReplyDelete
  34. Since Edward Feser holds to capital punishment, I guess this is a place where I can ask this question

    When is it okay to kill someone?
    As long as its justified, is it okay to feel good about it? Especially If there's nothing you can do about that feeling?
    Who deserves death? How can we tell?
    What about vigilance? Let's say I know someone who deserves death, and the justice system won't punish him for one reason or another, should I do it?

    ReplyDelete
    Replies
    1. I understand that this is angry bait, but are you familiar with courts, authority, legal processes and legitimacy?

      Delete
    2. All of these questions are dealt with in a fair and reasonable way in the book. You have good questions, Anonymous, and the book has good answers to them. So, it's an excellent fit: read the book.

      Delete
  35. How does Thomism specifically avoid the naturalistic fallacy?

    ReplyDelete
    Replies
    1. Short answer: Thomism does not share those dubious premises that naturalism supposes.

      Dr Feser wrote about it in Aquinas: A Beginner's Guide:

      " From the traditional Thomistic point of view, however, there simply is no
      “fact/value distinction” in the first place."


      ...

      "A gap between “fact” and “value” could exist only given a mechanistic-cum-nominalistic understanding of nature of the sort commonly taken for granted by modern philosophers, on which the world is devoid of any objective
      essences or natural ends. No such gap, and thus no “fallacy” of inferring normative
      conclusions from “purely factual” premises, can exist given an Aristotelian–Thomistic
      essentialist and teleological conception of the world."


      Feser has also noted in a post on here:

      "The possibility of such a “fallacy” arises only if we take for granted a modern mechanistic philosophy of nature, which, of course, the Neo-Scholastics did not."

      Delete
  36. What is your opinion of Pope Leo XIII's cocaine use and promotion?

    http://cocaine.org/popecoke.htm

    "His Holiness The Pope enjoyed the invigorating properties of coca wine. Leo XIII carried a personal hipflask to fortify himself in time of need. A grateful Pope awarded a Vatican gold medal to its distinguised orginator, the Corsican-born pharmacist and businessman Angelo Mariani. Mariani had a keen eye for the benefits of celebrity-endorsement."

    ReplyDelete
    Replies
    1. On first glance it looks like the wine in question had (and it isn't clear how much) coca plant in it which isn't the same as cocaine any more than margarine is napalm because they share a common ingredient.

      We don't have enough context to come within a light year of "cocaine use and promotion". Frankly, it's sounds like clickbait fodder.

      Delete
    2. To add to Scott's point, it's also notable that some of these claims come from taking Mariani's advertisements as if they were the gospel truth, which seems a particularly modern sort of gullibility. Leo XIII gave out lots of medals as tokens of thanks for lots of things, and Mariani was certainly not above stretching a little thing into a big thing to make better ad copy. Just as we don't really know much about the chemistry of coca wine, since it was banned before anyone really studied it properly, and so the most we can say is that it would probably have had at least some traces of cocaethylene in it, so also we can say little more about the connection to Pope Leo XIII than that Mariani probably did get some kind of letter of thanks for something, with a medal of some kind.

      Delete
    3. Since coca leaves were used in Coca Cola originally (until some time in the 1900s), it follows that a great many Americans imbibed coca chemicals including the ones that are extracted to produce cocaine. It does not follow that they got high from its use. The degree of difference in effects, (between using coca leaves and using cocaine) both from greatly concentrating the chemicals and from changing them in processing, is obviously very significant. Click-bait indeed.

      Delete
  37. Book Recommendations?

    Hi, all. I've spent this summer reading books regarding the philosophy of religion, and in particular the rational justifiability of Christianity.

    As a Christian, I am looking to gain further insight into the history of Christianity, the New-Atheist historical argument against Christianity (that religion "poisons everything" and an atheist future would be "utopia"), theology, philosophy, etcetera. So...insofar as rationally vindicating Christianity against secularism is concerned, would anyone have any book suggestions outside of the works which I already plan to read: those of the New Atheists, William Lane Craig, Dr. Feser, David Bently Hart, C.S. Lewis and Lydia McGrew?

    I plan on reading all of the New Atheist literature, so I can desensitize myself to the initial shock of disagreements and condescending insults, and know the atheist position hopefully as well as atheists themselves; but I also want to understand the more substantive debate around Christianity (i.e. the historicity of the Bible, replies to Bhart Erhman, clearing up theological misconceptions). I want to build a philosophically and theologically coherent worldview, and, going into my Junior year in high school, I see that I have a long way to go.

    So, really anything (books, essays, blogs, ect) that you think would help me pursue an informed worldview and to live a better life. What books do you wish you had read when you were younger? Thanks to anyone who read this, and especially those who may respond!

    ReplyDelete
    Replies
    1. you can visit the theism forum, they have some good resources.

      Delete
  38. I'm looking for a resource that develops Aristotle's argument in Physics book VI, to the effect that an infinite motion in a finite time is impossible. Can someone provide a recommendation? I'm having a hard time understanding it without reading Leibniz's argument against infinite wholes into the text.

    ReplyDelete
  39. Dr. Feser,

    What do you make of Jordan Peterson's strange conception of God as a sort of Darwinian thought pattern? I've been reading Maps of Meaning recently, and while I find the book intriguing, I'm a bit disturbed by the naturalistic explanations he gives for the evolution of the concept of God, and meaning itself, for that matter. I've always had an interest in similar thinkers like Carl Jung, Joseph Campbell, and Mircea Eliade, but their "theology" has shaken my faith far more than any New Atheist ever could. It's far easier to disagree with someone who says "God doesn't exist," than it is to disagree with someone who says, "Of course God exists; it just depends on what you mean by 'God.'"
    I ask this as a confused undergrad with no philosophical training. I'm just not sure how to go about navigating these thinkers' philosophical assumptions, so your insight would be much appreciated.

    ReplyDelete
    Replies
    1. I have been viewing quite a bit of Jordan Peterson's content. I'm not sure what Dr Feser would say in response, but here are my thoughts.

      The most important thing to note with respect to the existence of God is this: Peterson does not engage with classical natural theology, and so whatever he says in regard to the existence of God will always lack substance (in that respect). Insofar as Peterson, and other similar thinkers, do not substantively interact with actual classical natural theology - the very arguments that show God is real, shows there is one ground of all being etc. - then genuine theism is left untouched.

      No doubt one can describe the pragmatic benefits of theistic belief, the evolutionary benefits, existential implications, talk of God as a concept etc. Yet such talk is impoverished taken as a way to talk about whether God exists.

      To put it another way: the arguments for the existence of God, say, as defended by Dr Feser, are sound and powerful arguments, taking almost undeniable premises to their required conclusion: the existence of one pure actuality that has intellect, from which all things come, upon which all things depend for their very existence here and now etc. And the sorts of things Peterson says do nothing to undermine those conclusions.

      In the end, showing God is real is a matter of natural theology. While some may offer very interesting and well thought out ideas about God and belief, it has no bearing on the conclusions of natural theological arguments.

      Delete
  40. Is rejection of certain teachings of the second Vatican Council heresy?
    And how can the council itself reconcile many of its reachings with the prior ones? For example Lumen Gentium is in obvious contradiction with prior magisterium on salvation, and infallible one at that. How does one resolve this epistemic crisis?

    ReplyDelete
    Replies
    1. Claims that V-II conflicts with earlier doctrine tend to be overblown: there is a lot of blather, but a careful approach to the documents typically only resolves down to an extremely narrow set of statements, and as far as I know many careful traditionalists lay claim to 2 theses in V-II that are incompatible with Tradition. One of them is the thesis in Dignitatis Humanae in favor of religious freedom (even for heretics), taken by traditionalists to conflict with prior teaching that heretics are not free to believe their heresy nor free to teach it. The second is the thesis that even people who are not baptized may be saved if they are of good will, taken to conflict with the doctrine that outside the Church there is no salvation.

      Both of these theses, in V-II, are capable of being read with sufficient nuance that they do not conflict with the traditional teaching. For example, with regard to the second point: many of the formulations in Church history that refer to the damnation of those who are not baptized can clearly be understood to refer to those who, understanding the sound basis for the Church's claim to be the one pathway of salvation, STILL refuse to be baptized. At the same time, the doctrine of baptism by desire has expanded to at least theoretically address the situation of a person of good will who simply is never exposed to Christianity in their lives and therefore could not possibly receive formal baptism. Indeed, even as early and as traditional a source as the great Angelic Doctor (in the 13th century), taught that there are people who choose rightly and receive the grace of baptism even without overt (visible) Church action - and yet it remains true that (a) they receive this grace through the merits of the Church willed to it by Jesus as the Head of the Church, and (b) that by being in God's grace, become members of the Body of Christ that is the Church. So even if there is not a ceremony of baptism, a person can be a member of the Church and be saved.

      There is, also, a reconciliation for the Dignitatis Humanae issue as well.

      Since an Ecumenical Council's formal decrees are Church teaching documents, they must be read in a way that pushes you to take them in a sense that conforms with all prior doctrinal teaching. So any apparent contradiction must be assumed to be apparent only, and thus lead one to search for the "solution" that enables one to read the new in the light of the old and in conformity with the old.

      Delete
  41. Dr. Feser, I think I've found a flaw in Aquinas' Third Way. Here is my understanding of the Third Way, based on your book Aquinas:

    1. Some things are created and destroyed.
    2. Therefore, some things are such that it's possible for them to exist and possible for them not to exist.
    3. If it is possible for something not to exist, then there must be a time at which it will not exist.
    4. If all things are such that it's possible that they do not exist, then it is possible for there to be a time at which nothing exists.
    5. If it is possible for there to be a time at which nothing exists, then there must be a time at which nothing exists.

    Now if possibility and necessity have their usual meaning in modal logic, step 3 would be false, but you argue that they have a different meaning here: "By “possible not to be,” then, what Aquinas means is something like “having a tendency to stop existing,” “inherently transitory,” or “impermanent”; and by “necessary” he just means something that is not like this, something that is everlasting, permanent, or non-transitory. Thus there is no fallacy in his inference from “such-and-such is possible not to be” to “such-and-such at some time is not,” for this would follow given an Aristotelian understanding of the nature of material substances."

    But then if that's the meaning of possibility and necessity, then how in the world could you justify step 4? You say "[I]f it is even possible for every contingent thing to go out of existence together (which even Aquinas’s critic must concede), this possibility must actually come about." But Aquinas' critic need not concede that at all. If we adopt the usual meaning of possibility and necessity used in modal logic, then step 4 might be plausible. But if as you claim "possible not to be" refers to an inherent tendency to stop existing, then why must Aquinas' critic concede that all contingent things have an inherent tendency to stop existing together?

    A critic of Aquinas can easily maintain that each contingent thing has a tendency to stop existing, but all contingent things do not have a tendency to stop existing togethe.

    ReplyDelete
    Replies
    1. The first thing to note is that expositors and defenders of the Third Way do not all share this reading. There is disagreement over how it should be read, in light of certain possible historical sources from which Aquinas drew inspiration for the argument.

      But one thing remains: it must be read in light of possibility and necessity whereby possibility refers to an inherent incapacity of a thing to last forever and thus is transient; and necessity refers to a thing to be everlasting.

      An argument that can help one read the third way is found in the summa contra gentiles. It runs thus:

      We find in the world, furthermore, certain beings, those namely that are subject to generation and corruption, which can be and not-be. But what can be has a cause because, since it is equally related to two contraries, namely, being and non-being, it must be owing to some cause that being accrues to it. Now, as we have proved by the reasoning of Aristotle, one cannot proceed to infinity among causes. We must therefore posit something that is a necessary being. Every necessary being, however, either has the cause of its necessity in an outside source or, if it does not, it is necessary through itself. But one cannot proceed to infinity among necessary beings the cause of whose necessity lies in an outside source. We must therefore posit a first necessary being, which is necessary through itself. This is God, since, as we have shown, He is the first cause. God, therefore, is eternal, since whatever is necessary through itself is eternal.

      Brian Davies is one who views this as the most apt context through which to read the Third Way.I think reading the third way in light of this has its strengths.

      Nevertheless, Feser, along with many others, read the Third Way differently. While he goes over it in his book on Aquinas, he also outlines it in his paper Existential Inertia and the Five Ways. You can see a limited preview of it here:

      Existential Inertia and the Five Ways - Scroll to page 143.

      Also, ignore Stardusty, who commented below. He has demonstrated his ignorance on these matters. He clearly has as a chip on his shoulder, and refuses to substantively engage with the arguments - he seems to assume he can read a short paragraph summary offered by Aquinas, refuses to acknowledge Aquinas details the arguments in subsequent sections of his work, and ignores the fact that Aquinas subsequently shows precisely why the conclusion of the arguments are all the same one pure act and intellect from which all comes.

      Delete
  42. Keshav SrinivasanJuly 2, 2017 at 8:52 AM

    "Feser: [I]f it is even possible for every contingent thing to go out of existence together (which even Aquinas’s critic must concede),this possibility must actually come about."

    Aquinas and his followers frequently confuse a logical possibility, a logical necessity, a real possibility, a real necessity.


    --Aquinas fails in the 3rd way as he does in all his 5 arguments for the existence of God.

    "[I]f it is even possible for every contingent thing to go out of existence together (which even Aquinas’s critic must concede)"
    --I do not concede this. What does it mean for a contingent thing to go out of existence? What is a "thing" in this context? By "thing" do you mean a particular organization of constituent parts, say, a log, that is contingent upon growing in a tree and being cut and split by a human, and is then burned, and its parts are disbursed such that the organized object of a log no longer exists?

    If so then I deny any relevance to an argument for God since the constituents of the log did not cease to exist merely because the organized object we call a log no longer exists.

    Aquinas fails in the end as he always does, aside from his poor notions of logical and existential possibilities, with the last short sentence:

    "This all men speak of as God."
    This is a demonstrably false premise, which alone makes the argument unsound. Further, Aquinas does not even complete his argument for the existence of God, stopping short at the mere utterances of men.

    It would be a clear non sequitur to make the implied assertion of the existence of God from the mere utterances of men, thus rendering the argument for the existence of God logically invalid.

    ReplyDelete
  43. Is there a strict demonstration of the existence of tele/final causes? Something along the lines of 'a denial leads to a reductio'?

    ReplyDelete
    Replies
    1. Well, one who denies final causality would need to tell us why an agent inclines always, unless impeded, to a particular end. And if they answer: "because that's its nature" - then they really do affirm final causality. As Aquinas put it "to intend this [an end] is nothing else than to have a natural inclination to something."

      And if they deny that anything acts toward a particular end, then kindly invite them to observe nature.

      As for arguments, Feser offers several arguments in his Scholastic Metaphysics p.92-104. To summarize: chance is not a serious alternative; nor is efficient causality alone. Neither are appeals to laws of nature.

      Many who deny finality often simply do not know what it is. And the corollary of final causality is clearly what motivates many people to desperately deny it.

      Delete
    2. JasonJuly 2, 2017 at 6:56 PM

      " As Aquinas put it "to intend this [an end] is nothing else than to have a natural inclination to something."

      --Ok, so stuff generally ends up doing what stuff is inclined to do.

      That is true in the sense a tautology is true, that is to say, trivially.

      But Aquinas says a great deal more than that in the Fifth Way.

      "so as to obtain the best result. ... designedly, do they achieve their end."

      An end is not merely the result of incremental inclinations, rather, it is "the best result" "designedly".

      A design is a preexisting plan. The end is not merely the result of an inclination, in this claim, rather a plan for a best result.

      But where is this plan, this best result evident in nature? I don't see one, rather, I see things going in chaotic directions, often to the suffering and destruction of much that seems very good to me, and therefor not a best end by design in any evident sense.

      "Now whatever lacks intelligence cannot move towards an end,"
      --Indeed, an end is not achieved by mere unintelligent inclinations, because an end is a plan, a plan for a best result, that requires an intelligence, and thus has a purpose in the mind of the designer.

      So no, an end cannot be accounted for by a naturalistic inclination, and thus the notion that nature works toward an end is illusory.

      Delete
    3. I know this is futile, but it isn't trivial at all. The fact things are inclined to act in such and such a way under such and such conditions is an explanation for causal regularity. The modern/Humean dismissal of final causes leaves one with no explanation for such regularity.

      Delete
    4. Anonymous,

      That was exactly my thought, as well. The denial of this supposedly trivial claim is a major component of most modern theories of causation.

      Delete
  44. iwpoe, is not interior evidence sufficient? Don't you observe yourself acting for a purpose?

    This would not be a strict demonstration, I grant. But neither are fundamental principles, which are known with certainty. Nor is the observation of change, which is perceived in its proper mode of assurance.

    ReplyDelete
    Replies
    1. @Tony,


      Are there any actually strict demonstrations of final causality and the existence of God though?

      And what fundamental principles which are known with absolute certainty do not have a strict demonstration exactly?

      Delete
  45. Hi Dr. Feser,

    How would you argue for scholastic metaphysics considering contrastive underdetermination, and holistic underdetermination?

    ReplyDelete
  46. @Tony,


    Are there any actually strict demonstrations of final causality and the existence of God though?

    And what fundamental principles which are known with absolute certainty do not have a strict demonstration exactly?

    ReplyDelete
    Replies
    1. Joe, I guess my point was not clear enough. When you ask for "strict demonstration" you assume that the point for which you are asking is something capable of demonstration, i.e. something for which demonstration is a suitable proceeding.

      But not everything is demonstrable (if we mean by it what is meant in A-T circles). For, demonstration consists in using true and certain principles (in the form of propositions) to reason to conclusions. It is not the correct procedure for grasping the first principles, it assumes them. Since drawing out conclusions must necessarily rely on premises, then the FIRST premises must necessarily be known not through demonstration but through some other mode of apprehension. Either demonstration rests on premises that are not demonstrable, or you get circular arguments that don't rest on ANYTHING.

      One such that is not demonstrable is the principle of non-contradiction: a thing cannot both be and not be at the same time in the same respect. Nor does it need to be demonstrated by a process of reasoning from premises, it is known with certainty as soon as one knows the meaning of the terms. Another is a principle of math (and natural science), a whole is quantitatively great than a proper part of that whole (colloquially abbreviated to: the whole is greater than the part). It is a mistake to think that these principles need additional truths to establish their validity, beyond knowing the meaning of the terms. Thus they neither need demonstration, to be known, nor (properly speaking) can they be known through demonstration, because their truths do not rest on more foundational truths than just the meaning of the terms.

      But there are other realities that are known in their proper mode of apprehension that are not the result of propositional reasoning. Two examples: you don't need to demonstrate "red", you have immediate direct experience of it. Nobody need establish the experience "red" to you "with certainty", and it would be silly to think that a demonstration could be more certain about it than your own direct experience. The other type is the apprehension of types, kinds, natures, the what-sort-it-is-ness of different kinds of things. When you first experience a cat, you don't know much of what-it-is. By the time you experience several cats of different colors, sizes, doing many different things, etc, you have conceived an idea, a concept, a type, to which the word "cat" will correspond. It is nonsensical to think that one demonstrates the meaning of "cat", for demonstration RELIES on the use of premises (which are propositions) for which you already know the meaning of the terms. You cannot demonstrate (in the A-T sense) the meaning of "cat", that is a step prior to demonstrating about cat.

      Delete
    2. So my initial point was: that there is final causality does not need to be "demonstrated", you have direct, immediate experience of it. It can be pointed out, as I did, but it cannot be proven by premises.

      It also bears noting that we should have a respect for having different kinds of certainty for different kinds of things known. The kind of certainty we have of "red" is one sort, the kind of certainty we have of "that there IS red" is another sort, and the kind of certainty that "things are red in virtue of reflecting light in the 650 angstrom wavelength" is a third kind. I am not merely saying that we have different degree of certainty, but that we have a different sort of certainty of these different modes of knowing, so that we should not expect to compare them directly with each other. They each have their own type of certainty appropriate to them.

      Nor is ANY of our certainty like to God's certainty. We are not gods, with absolute knowledge of everything. If "to know with certainty" meant to know the way God knows, then it would be right to say that we don't know certainly. But that is not the appropriate meaning for the term. We know SOME things with the appropriate human mode of certainty for that kind of thing, and others we have lesser degrees of certainty. So, in examining a proposition, say, the Pythagorean theorem, a student who has just started geometry can have a strong opinion that it is true, because he relies on and trusts his teacher who declares that it is provable. The student is fairly confident of its truth, and his confidence is subject to degrees of more and less depending on his trust in the teacher - and so some students may be very confident of the claim, others less. For all of the students, the kind of confidence they have is that of opinion, but opinion is not the highest sort of confidence applicable to that proposition. But the teacher (assuming he is a competent and thoroughly trained mathematician) DOES know the demonstration that establishes the truth with certainty. The teacher, then, has a different kind of confidence in it than the students. And, among the different teachers who have understood the demonstration, they all have the same kind of confidence, that of demonstration: to them it is certain in the mode proper to propositions capable of demonstration.

      But that mode is quite different from the certainty that pertains to other sorts of apprehension, such as for propositions that are not capable of demonstration (e.g. "I prefer chocolate over strawberry, today"), and things known that are not even propositional, such as the essence of cats.

      But since God knows in a manner and mode that is higher than that of reasoning step-by-step from premises to conclusions (or any of the other distinctly human modes of knowing), the kind of certainty that is proper to God's knowing is a different sort than the kind of certainty that is proper to human knowing. So don't expect to apply to humans the sort of "absolute" certainty that only God could have. Demonstration provides certainty of a certain type, for propositions for which demonstration is suited, but it is not the only sort of certainty.

      Delete
    3. @Tony,

      ''One such that is not demonstrable is the principle of non-contradiction: a thing cannot both be and not be at the same time in the same respect. Nor does it need to be demonstrated by a process of reasoning from premises, it is known with certainty as soon as one knows the meaning of the terms. ''


      The principle of non-contradiction cannot be demonstrated, I agree. But it is known with absolute certainty because to deny the principle you must assume it, thereby making a denial of the principle incoherent and impossible.

      This would then be an argument by retortion, or a reductio ad absurdum, which is also a way which we can know things with certainty, if not an actual way of demonstrating things as well.

      ''But there are other realities that are known in their proper mode of apprehension that are not the result of propositional reasoning. Two examples: you don't need to demonstrate "red", you have immediate direct experience of it. Nobody need establish the experience "red" to you "with certainty", and it would be silly to think that a demonstration could be more certain about it than your own direct experience. ''


      Well, that would be similar to the Cogito then, don't you think?

      The Cogito basically shows that you know your own existence with absolute certainty, which is different from the way you know other things.

      Of course, classical philosophy completely rejects the Cartesian representationalist view of perception and experience, which means that skepticism about the external world or anything else cannot even get off of the ground.

      As such, it seems that the reality of the external world under classical philosophy is also known with a great deal of certainty as well. Heck, this is what is called a direct realist apporach.

      And direct realism sure gives people the ability to say that they know the external world in a direct manner close to, if not equal to, the certainty that they know of their own existence.

      Delete
    4. @Tony,

      ''The other type is the apprehension of types, kinds, natures, the what-sort-it-is-ness of different kinds of things. When you first experience a cat, you don't know much of what-it-is. By the time you experience several cats of different colors, sizes, doing many different things, etc, you have conceived an idea, a concept, a type, to which the word "cat" will correspond. It is nonsensical to think that one demonstrates the meaning of "cat", for demonstration RELIES on the use of premises (which are propositions) for which you already know the meaning of the terms. You cannot demonstrate (in the A-T sense) the meaning of "cat", that is a step prior to demonstrating about cat.''


      I guess one knows the meaning of cat via a retorsion argument, or a reductio of a similar type to the Cogito.

      So in other words, in order for demonstrations to exist, the meaning of words and propositions needs to exist in a definite manner in the first place.

      In other words, the meaning of words and the essences of natures is more certain than any demonstration, deductive or inductive.

      Hmmm...something tells me we could make an argument for the existence of God based on logic and existence itself, though I'm not sure if Aquinas would contend with that.


      '' that there is final causality does not need to be "demonstrated", you have direct, immediate experience of it. It can be pointed out, as I did, but it cannot be proven by premises. ''

      So final causality is more certain than demonstration in the same way the meaning of the word cat is more certain to someone than any demonstration could be. Which means it has more certainty then.

      ''Nor is ANY of our certainty like to God's certainty. We are not gods, with absolute knowledge of everything.''

      Wouldn't the knowledge of one's own existence, the Cogito, still be an example of absolute certainty though?Also, this basically means we won't get absolute God-like certainty until we reach the Beatific Vision, right?

      Delete
    5. This would then be an argument by retortion, or a reductio ad absurdum, which is also a way which we can know things with certainty, if not an actual way of demonstrating things as well.

      I take the retortion as being an argument known to validate the conclusion based on the principle of non-contradiction. It is not MORE BASIC than the principle, and using the argument is logically comparable to pointing to a red thing and saying "that's an example of red". It is a manner of exhibiting to the mind the truth of the proposition by another way of considering the same truth at the same 'level' of knowledge, not by a truth logically and really prior to the principle.

      The Cogito basically shows that you know your own existence with absolute certainty, which is different from the way you know other things.

      I think that with St. Thomas, a person's awareness of the existence of the self, which you refer to as the "Cogito", is via direct immediate experience, but is not the primary experience: it requires first other experiences of other things (like sensing sensible objects, and experiencing pain and pleasure, and desires for things), and then the reflection on those gives rise to the direct experience of self as such. So, that implies that the Cogito is not prior to experiences given through senses, interior and exterior, it relies on those, and thus the certainty of the Cogito is at least in some sense no greater than the certainty of those other experiences.

      Delete
    6. @Tony,

      In other words, Aquinas destroys radical skepticism of the external world and other common sense beliefs in a way that is beyond just all reasonable doubt, but close to all possible doubt as well?

      If that is the case, then I think we could easily establish classical philosophy as being objectively more reasonable than modern philosophy without having to get our hands dirty with philosophical argument then.

      But another thing is:

      Can humans know ANYTHING with absolute certainty?Or is the certainty always going to be close to absolute, but not a 100% ?

      The principle of non-contradiction seems like it could be something that is known with absolute certainty, and reality of the external world follows as a close second then, if not even closer in terms of certainty.

      Delete
    7. @Tony,

      Also, considering this is an open thread, another thing I would like to ask you is this.

      A certain objection could be made to the idea that God is Being itself.

      According to Parmenides, being implies form, even existence itself as a concept has form, and thus this would put a limit on God if he were Being itself.

      A similar objection was made by Urs von Balthasar in his critique of Thomists who say that God is just Being.

      And even the Eastern Orthodox church holds that the divine essence is beyond being and non-being, consciousness and non-conscioussness, at least from the interpretations of Gregiry Palamas that I've seen.

      What would be the Thomistic response to the objection that God being Being itself implies form which implies limit vis-a-vis Parmenides?

      Delete
    8. Can humans know ANYTHING with absolute certainty?Or is the certainty always going to be close to absolute, but not a 100% ?

      The principle of non-contradiction seems like it could be something that is known with absolute certainty, and reality of the external world follows as a close second then, if not even closer in terms of certainty.


      JoeD, I am not as up on Thomistic epistemology as some, so take this with a grain of salt, but my understanding is that with A-T epistemology (and their understanding of the human mind and senses), ALL of the self-evident principles are in the same category of certainty as the principle of non-contradiction. That is, the mind holds them in the same mode of apprehension: it perceives the truth in the very operation of understanding the terms of the proposition. They are ALL "certain" in the proper sense of the term, as applicable to propositions. When one properly understands the terms, they are not doubtable. However, some of them seem more foundational than others, in that denial of the less foundational ones does not imply denial of as many others. One could claim to doubt "the whole is greater than the part" without obviously doubting non-contradiction, for example. But this does not mean that "the whole is greater than the part" is "proven" by the principle of non-contradiction, just that the latter is connected to every truth because it touches all being (and thus all truth because being is convertible with truth).

      The Cogito and the certainty we have of sense are, I think, also in the same category of certainty, though I think a different term is more proper, since it is not propositional truth. Aristotle says that we are incapable of being mistaken with respect to the primary object of sense: e.g. the visual faculty cannot be mistaken as to a color perceived. (However, the mind can be mistaken as to asserting the existence of a certain object having the color thus perceived). I would be fairly cautious about how far that 'certainty' takes us: since it is in a different category of certainty as that of knowing principles, I would not call it more or less. It is, rather, the highest form of assurance in that category, which is (by our make-up) designed for a certain sort of knowledge and not for other knowledge.

      It seems to me that the assurance we have of the existence of external objects comes especially from the coincidence of multiple sources of sense impressions: the object looks round, and feels round, and smells like an apple, and tastes like an apple, and thumps like an apple when you strike it. Aristotle makes an analysis of common sensibles (like the perception of movement, which depends on the primary sensible color, but also depends on the memory of the prior arrangement of colors, and other factors), which probably is the ground for our assurance of the independence of external objects. I don't recall the details. But it is derivative, I think, from other perception. While we can be mistaken about the reality of a given object, the proposition that we could be mistaken about ALL of them is more problematic, but again I don't recall the details.

      Delete
    9. So, what I am getting from this, is that radical skepticism of the senses is simply a non-starter for A-T.

      Because A-T epistemology easily renders any and all skeptical scenarios moot by virtue of common-sense and the certainty that common-sense really and actually gives, right?

      Also, how much certainty can we get from the A-T arguments for the existence of God?

      I recall Feser saying in one of his books that the metaphysical arguments establish a certainty beyond all possible doubt, due to the nature of the certainty such a demonstration carries.

      Now I am not sure how Feser refutes the idea of brute facts, as it seems Ed just gives a retorsion argument that makes a denial of the PSR extremely absurd to the point that any reason one could give to postulate brute facts is incoherent because it could as well not be a good reason but just an unjustified brute fact.

      Any thoughts?

      Delete
  47. We know SOME things with the appropriate human mode of certainty for that kind of thing, and others we have lesser degrees of certainty.

    Should have been "lesser degrees of confidence".

    ReplyDelete